Criminal Flashcards

1
Q

You are prosecuting Emma, a police officer, for manslaughter after a suspect, Nicholas, died in custody while Emma was restraining him. Emma pleads not guilty. Her defence is that her use of force was reasonable in the circumstances because Nicholas was violent. Five years ago, Emma was investigated for police misconduct after another suspect, Graham, made an accusation that she used excessive force when arresting him.
You apply to admit this previous misconduct, but the judge rules that as it was a single incident from five years ago, it is not relevant to an important matter in issue between the prosecution and defence.
During the trial, while she is being cross-examined, Emma says, “I’ve never been accused of anything like this before. I have an exemplary service record.”
What would be the most appropriate next step to take?
[A] Cross-examine Emma about the previous allegation of police misconduct made by Graham to correct the false impression she has given.
[B] Apply to cross-examine Emma about the previous allegation of police misconduct made by Graham to correct the false impression she has given.
[C] Apply to cross-examine Emma about the previous allegation of police misconduct made by Graham, to show that Emma has a propensity to use excessive force.
[D] Cross-examine Emma about the previous allegation of police misconduct made by Graham, to show that Emma has a propensity to use excessive force

A

ANSWER
[B] Apply to cross-examine Emma about the previous allegation of police misconduct made by Graham to correct the false impression she has given.
[A] This is not the best answer as although Emma has denied being accused of anything like this before, the prosecution would need to apply to the Judge under s101(1)(f) before cross-examination, as the Judge must rule on whether Emma has made a false impression under gateway (f) and what evidence may be admitted to correct that false impression. Evidence admitted under gateway (f) is limited to evidence correcting a false impression (s. 105(6)) and the judge would have to rule on what could and could not be referred to here. You would not be able to simply cross-examine her without making an application first.
[B] This is the best answer because it would be the judge who rules on this issue (see the answer to [A] above. Though the judge previously ruled this evidence was excluded under s101(1)(d), it may now be potentially admissible under gateway (f) because Emma is suggesting there have been no allegations of excessive force against her in the past where there has been. However, it is a matter for the Judge whether she has made a false impression.
[C] This is not the best answer as although there is a potential argument that this showed a propensity to use excessive force, the Judge has already ruled that as it was a single matter and five years ago, it was not admissible. The comments of the defendant have not changed this position, so there are no new grounds to make the application again. Therefore, this is not the best ground to apply under.
[D] This is not the best answer for the reasons set out in [C] above. Also, you would need to apply to cross-examine, rather than simply cross-examine without an application.

How well did you know this?
1
Not at all
2
3
4
5
Perfectly
2
Q

You are prosecuting Salvatore, who is on trial for s.20 GBH against a former girlfriend who suffered two lost teeth during the attack. Towards the end of the trial the complainant asks you, in the event that Salvatore is convicted, what factors the court will take into account when passing sentence in the context of the impact of the incident on her.
What is the correct advice to give her?
[A] The complainant can provide the court with a victim personal statement, which can make suggestions about the appropriate type or length of sentence Salvatore should receive because of the impact of the offence.
[B] The complainant can provide the court with a victim personal statement, which will ensure that the court will consider the complainant’s evidence about the personal impact of the offence.
[C] The complainant is obliged to assist the court by providing a victim personal statement, which is necessary for the court to consider aggravating and mitigating factors when passing sentence.
[D] The complainant is not obliged to provide a victim personal statement but if she does not, a judge may infer from the absence of a victim personal statement an absence of harm.

A

ANSWER
[B] The complainant can provide the court with a victim personal statement, which will ensure that the court will consider the complainant’s evidence about the personal impact of the offence.
[A] This is wrong and not what the VPS is for. According to Perkins [2013] EWCA Crim 323, [2013] 2 Cr App R (S) 72 (460) VPS’s “provide a practical way of ensuring that the Court will consider the evidence of the victim about the personal impact of the offence. The process is not an opportunity for the victim to suggest, or discuss, the type or level of sentence to be imposed. The distinction is important.”
[B] This is the correct answer - see [A].
[C] This is wrong. The complainant is not obliged to make a VPS: “Victims must be provided with information which makes it clear that they may make a statement but are under no obligation to do so.”
[D] This is wrong. The judge is not to assume that there has been no impact if no VPS is provided: “A judge must not assume that the absence of a victim personal statement indicates an absence of harm” (Chall [2019] EWCA Crim 865).

How well did you know this?
1
Not at all
2
3
4
5
Perfectly
3
Q

You represent Barbara, who is being tried summarily for an offence of criminal damage where the value is in excess of £5,000. The magistrates find Barbara guilty, impose a fine and order her to pay the prosecution costs. Following the hearing, Barbara tells you she wishes to appeal both her conviction and sentence to the Crown Court. She is concerned about any negative consequences of pursuing the appeal.
What is the correct advice to give Barbara in relation to the Crown Court’s powers if she is unsuccessful in her appeal?
[A] It can order Barbara to pay the prosecution costs of the appeal and can increase her sentence to the maximum that the magistrates’ court could have passed.
[B] It can order Barbara to pay the prosecution costs of the appeal but cannot increase the sentence passed by the magistrates at the conclusion of her trial.
[C] It can order Barbara to pay the prosecution costs of the appeal and can increase her sentence to the maximum that it could have passed if she had been tried on indictment.
[D] It cannot order Barbara to pay further prosecution costs but can increase her sentence to the maximum that the magistrates’ court could have passed.

A

ANSWER
[A] It can order Barbara to pay the prosecution costs of the appeal and can increase her sentence to the maximum that the magistrates’ court could have passed.
[A] is the right answer because it encapsulates s48(4) Senior Courts Act 1981 and the rules in relation to prosecution costs covered in CrimPR 45.
[B] is wrong because they can increase her sentence.
[C] is wrong because they can only increase her sentence to the maximum the magistrates’ court could have imposed.
[D] is wrong because they can order her to pay the prosecution’s costs on the appeal irrespective of whether they ordered costs at the original trial.

How well did you know this?
1
Not at all
2
3
4
5
Perfectly
4
Q

You are defending Gordon in the magistrates’ court. He has pleaded not guilty. At a pre-trial hearing, the prosecutor applies for the witness statement of Brandon, a key prosecution witness, to be admitted as hearsay evidence because Brandon cannot be located. The District Judge rules that the evidence can be admitted as hearsay at the trial. On the day of trial, you are informed by the prosecutor that Brandon has been located and is available to attend the hearing, but that she does not intend to call him as a live witness. You ask the prosecutor to call Brandon to give live evidence, but she refuses, insisting there is no obligation to do so because his witness statement has already been ruled as admissible hearsay.
What is the most appropriate action for you to take?
[A] Apply to the magistrates to discharge the pre-trial ruling allowing Brandon’s evidence to be admitted as hearsay.
[B] Inform the magistrates that Brandon is available so that they can consider whether to discharge the pre-trial ruling of their own motion.
[C] Seek the prosecutor’s agreement for Brandon to be called to give live evidence.
[D] Proceed to trial as planned, allowing Brandon’s evidence to be admitted as hearsay.

A

ANSWER
[A] Apply to the magistrates to discharge the pre-trial ruling allowing Brandon’s evidence to be admitted as hearsay.
[A] The best answer. There has been a material change of circumstances since the pre-trial ruling. Given the prosecutor’s refusal to voluntarily call Brandon the best approach is to apply for the magistrates to discharge the pre-trial ruling under s.8B(3) MCA 1980.
[B, C, D] These are not the best answers. Whilst all technically possible they do not best answer the question. There has been a material change of circumstances since the pre-trial ruling. Given the prosecutor’s refusal to voluntarily call Brandon the best approach is to apply to the magistrates to discharge the pre-trial ruling under s.8B(3) MCA 1980.

How well did you know this?
1
Not at all
2
3
4
5
Perfectly
5
Q

Danna is convicted in the magistrates’ court of causing £100 worth of criminal damage.
What is the maximum custodial sentence that the magistrates can impose?
[A] Three months
[B] Six months
[C] Nine months
[D] Twelve months

A

ANSWER
[A] Three months
[A] is correct – as the value of the damage is below £5,000, the maximum sentence that can be imposed is three months. Therefore [B], [C] and [D] are wrong.

How well did you know this?
1
Not at all
2
3
4
5
Perfectly
6
Q

You prosecute Harrison (aged 16) in the youth court. He has pleaded not guilty to charges of robbery and having a bladed article in a public place (an either-way offence).
On the night of the incident, the complainant, Joseph (aged 19), gave a witness statement to the police in which he alleged that Harrison threatened to stab him if he did not hand over his mobile phone.
Two weeks before the trial, Joseph provided a further statement to the police stating that everything in his first statement is true, but he is no longer prepared to attend court and give evidence at trial, as he is fearful of reprisals from Harrison and his friends, who live in the same area.
Joseph’s further statement is served on Harrison’s solicitors. At a pre-trial review hearing, Harrison’s solicitor applies for the matters against Harrison to be discontinued on the basis that the prosecution cannot secure Joseph’s attendance at trial.
How should you respond to the defence application?
[A] Offer no evidence, as it is not in the interests of justice to secure Joseph’s attendance at trial in light of his further witness statement.
[B] Offer no evidence, as Joseph is unlikely to be able to give material evidence at trial as there is now a question over his credibility as a reliable witness.
[C] Apply for a witness summons, as Joseph is likely to be able to give material evidence at trial and it is in the interests of justice to issue such a summons.
[D] Apply for a witness summons due to the seriousness of the allegation, and the court’s duty to protect the public from harm in cases involving vulnerable witnesses.

A

ANSWER
[C] Apply for a witness summons, as Joseph is likely to be able to give material evidence at trial and it is in the interests of justice to issue such a summons.
[A] Is wrong. Joseph is likely to be able to give material evidence and it is likely to be in the interests of justice for the court to issue a witness summons; Joseph’s further statement makes clear that the original allegation is true, and it is only the fear of reprisals which has led to Joseph withdrawing support for the prosecution as opposed to his original allegation being untrue. An answer which suggested that this is the correct approach would not be applying the test in MCA 1980 s.97 correctly.
[B] is wrong. Joseph is likely to be able to give material evidence for the purposes of a summary trial; Joseph’s statement makes clear that the original allegation was true. See explanation to [A] also.
[C] is correct. In light of the contents of both Joseph’s original statement and further statement, he is likely to be able to give material evidence for the purposes of a summary trial and given the serious nature of the allegation, the court is likely to conclude that it is in the interests of justice to issue a witness summons. This is the correct application of the test set out in s.97.
[D] is wrong as it is not the correct application of the test which the court must apply in order to determine whether to issue a witness summons.

How well did you know this?
1
Not at all
2
3
4
5
Perfectly
7
Q

You prosecute Gabriel, who is on trial in the Crown Court for theft. It is alleged that Gabriel was seen by his colleague, Marian, entering their boss’s empty office one lunchtime and taking a wallet which had been left on the desk. The wallet contained a quantity of bank notes.
The only other evidence is that 30 minutes after it was taken, the empty wallet was found in a bin close to Gabriel’s desk in his office, which is on a different floor of the building.
When giving evidence, Marian accepted that she had only started working in Gabriel’s office the week before the incident. She explained how she had been sitting at her own desk, a few metres away from her boss’s office, from where she witnessed the theft. In cross-examination she admitted that the offender had his back to her but she did catch a glimpse of his face as he left the office, at which point she saw him putting something into his inside jacket pocket. When asked how sure she was that the offender was Gabriel she replied, “It’s difficult, more than 90% certain.”
At the close of the prosecution case, defence counsel makes a submission of no case to answer.
What is your best response as to whether the case can safely be left to the jury?
[A] It can, provided that a full Turnbull direction is given in relation to the identification evidence during summing up.
[B] It can, because the identification evidence is supported by other reliable evidence.
[C] It can, provided that the jury are directed that they must not convict Gabriel based on the identification evidence alone.
[D] It can, because Marian’s identification is based on recognition, which is more reliable than identification evidence alone.

A

ANSWER
[B] It can, because the identification evidence is supported by other reliable evidence.
[A] is not the best submission. Although a Turnbull direction will undoubtedly have to be given in the summing up, it is not sufficient in this case. “A defendant cannot properly be convicted on qualified identification evidence alone”. Thus, to avoid a successful submission of no case to answer the prosecution must draw attention to the other reliable evidence in the case, namely the finding of the wallet.
[B] is the best submission. “…as with other kinds of weak identification evidence, a qualified identification may have a legitimate role to play alongside other, more reliable, evidence.” The best submission for the Crown to make in these circumstances is that a case based on qualified identification evidence can safely be left to the jury when it is supported by other reliable evidence, namely the finding of the wallet.
[C] is not the best submission. Again, this might be an attractive submission but there is no obligation on the judge to give this kind of warning. “Where a judge decides that the identification evidence in a given case is of such poor quality that he would not have left the case to the jury in the absence of supporting evidence, there is no obligation on him to warn the jury that they should not convict on the basis of the evidence of identification alone, should they reject the supporting evidence.”
[D] is not the best submission. Whilst possible, the facts of this scenario do not go far enough to suggest clear recognition.

How well did you know this?
1
Not at all
2
3
4
5
Perfectly
8
Q

You are prosecuting Karl, who is charged with dangerous driving (an either-way offence). Susie saw Karl’s car being driven at speed and crashing into a parked car before driving off. Susie wrote the car’s registration number on a piece of paper and read it out to a police officer, who then recorded it on a police incident report. CCTV is available, which shows the car being driven dangerously, but does not show the registration plate. Karl disputes that his car was involved and asserts that Susie recorded the car’s registration number incorrectly.
When Susie gives a statement a few days later, she has lost the piece of paper with the registration number on it. In her statement, Susie describes the driving. She also states that she gave details of the car registration number to the police but can no longer recall what it was. At trial, you wish to adduce the police incident report with the car registration number on it.
What is your best course of action to ensure that the report is adduced?
[A] Argue that the police incident report is admissible. It is not hearsay as Susie did not intend to cause anyone to act on the information.
[B] Argue that it is admissible hearsay under CJA 2003 s.117.
[C] Argue that it is admissible hearsay under CJA 2003 s.114(1)(d).
[D] Seek agreement from the defence for the police incident report to be admitted in evidence.

A

ANSWER
[B] Argue that it is admissible hearsay under CJA 2003 s.117.
[A] is not the best answer as when Susie wrote it down and then gave the information to the police, it is clearly arguable that this was done in order for someone to act upon it.
[B] is the best answer as the prosecution can argue for it to be admitted under s.117 of the CJA 2003 on the basis that it is a document, created or received by a person in the course of a profession, and the person (Susie) had personal knowledge of the matters contained in it. As Susie read out the registration number and supplied the information to the police at the time of an offence, it is arguable that it was prepared in contemplation of criminal proceedings or investigation (s.117(4)). If so, s.117(5)(b) is satisfied if it can be shown that the relevant person (Susie) cannot reasonably be expected to have any recollection of the matters dealt with in the statement having regard to the length of time since she supplied it (and she has said she cannot).
[C] is not the best answer as whilst the prosecution could argue that it is admissible under s.114(1)(d), the primary application should be under s.117 as the police incident report falls under this heading as a business document. The authorities indicate that the CJA 2003, s.114(1)(d) is to be applied with caution - s.114 has been referred to in terms of a ‘hierarchy’ of exceptions; s.114(1)(d) should not be used to circumvent requirements of other gateways further up the hierarchy.
[D] is not the best answer as Karl does not accept this is right and therefore the defence are most unlikely to agree to this evidence being adduced under s.114(1)(c).

How well did you know this?
1
Not at all
2
3
4
5
Perfectly
9
Q

Two months ago, Errol was released from a 16-month prison sentence for an offence of burglary. Shortly after his release, he began a relationship with Sarah and is now living with her at her parents’ house in Birmingham. Last night, Errol was arrested at this address for a further offence of burglary. It is alleged that seven days ago, he broke into a house in Bristol. It was during the day and the tenant was not on the premises at the time.
Errol accepts that he was in Bristol on the day of the burglary as he had travelled from Birmingham with friends to attend a football match. However, he denies the current allegation. If convicted, he is likely to receive a sentence in excess of the 16 months received for his last offence. The Court is considering conditional bail.
Which of the following conditions is the court most likely to impose on Errol?
[A] To observe a daily curfew between 20:00 and 07:00.
[B] To report to his local police station daily.
[C] A condition not to enter any Bristol postcode.
[D] A condition not to contact, directly or indirectly, the tenant of the property in Bristol.

A

ANSWER
[B] To report to his local police station daily.
[A] Is not the best answer. A curfew condition is unlikely to be imposed as this was a daytime burglary and there is no suggestion that he has a propensity to commit burglaries at a particular time.
[B] Is the best answer. A condition of bail can be imposed if it necessary to ensure, amongst other things a surrender to custody. Here Errol’s community ties are very weak (he is living in the house of the parents of his girlfriend, who he met since his release from prison two months ago. Prior to that he was in custody). In addition, he has a motive to abscond given the likely sentence on conviction. He is clearly a serious flight risk and so daily reporting would ensure the local police knew where he was.
[C] and [D] are not the best answers. The conditions in [C] and [D] might be appropriate to protect the tenant if he was likely to be subject to witness interference or if the burglar might strike again. However, interference is unlikely as the witness is not contentious; and there is no evidence that Errol is likely to revisit.

How well did you know this?
1
Not at all
2
3
4
5
Perfectly
10
Q

You are prosecuting Max. He has pleaded not guilty to common assault on Sienna, who alleges Max punched her after their cars were involved in a road traffic incident and Max then drove off. The details of the car in which Max drove off led the police to him.
Sienna made a witness statement a few hours after the incident. Her witness statement included the registration number, make and model of the car in which Max drove off. You wish to adduce these details about the car in court in order to help establish that Max was the perpetrator.
At trial, Sienna reads her witness statement before going into court. She then gives evidence. During her evidence-in-chief she remembers the incident very well but when you ask her to describe the car her attacker was driving, she says she cannot remember the make, model or registration number. She says that she wrote down these details in her statement as best as she could remember them shortly after the incident and when her memory of the details was significantly better.
What is your best course of action?
[A] Ask Sienna if she would like to withdraw from the courtroom for another opportunity to read her witness statement.
[B] Make an application under CJA 2003 s.139 for Sienna to be permitted to refresh her memory from her witness statement in the course of giving her evidence.
[C] Make an application for the details of the car to be admitted as hearsay under CJA 2003 s.114(1)(d).
[D] Make an application for the details of the car to be admitted as a previous statement under CJA 2003 s.120.

A

ANSWER
[B] Make an application under CJA 2003 s.139 for Sienna to be permitted to refresh her memory from her witness statement in the course of giving her evidence.
[A] Not the best answer. Although it is usual for a witness to refresh their memory outside court before giving evidence, there is no need for mid-evidence memory refreshing to be done outside court and indeed s.139 CJA 2003, which is satisfied here, does not envisage it.
[B] The best answer. The conditions are satisfied and given the witness’s very limited difficulty there is no need for any other application.
[C] Not the best answer. An uncertain and complicated route, and unnecessary as the witness can give the evidence live once she has refreshed her memory.
[D] Not the best answer. Technically possible in order to adduce Sienna’s statement as evidence of the matters stated and to show consistency but these aims are not appropriate or desirable where, as here, she can give live evidence perfectly well with assistance from s.139.

How well did you know this?
1
Not at all
2
3
4
5
Perfectly
11
Q

Helen (aged 17) and Susan (aged 18) are jointly charged with theft. It is alleged that Helen played the major role in the offence. Both defendants have numerous previous convictions for theft. They appear in the magistrates’ court for plea before venue and allocation. Susan’s solicitor has told you that Susan will be pleading not guilty and electing Crown Court trial. You represent Helen. She intends to plead not guilty and asks you in which court will her trial take place.
What advice should you give to Helen?
[A] If the magistrates accept jurisdiction, Helen should elect Crown Court trial as she will have a better chance of acquittal in front of a jury.
[B] Due to Helen’s age, it is unlikely that the magistrates will consider it is in the interests of justice for her to be tried at the Crown Court and therefore her case will be remitted to the youth court for trial.
[C] Due to Helen’s age, it is unlikely that the magistrates will consider it is in the interests of justice for her to be tried at the Crown Court and therefore she will stay in the magistrates’ court for trial.
[D] It is likely that the magistrates will consider it is in the interests of justice for Helen to be tried at the Crown Court.

A

ANSWER
[D] It is likely that the magistrates will consider it is in the interests of justice for Helen to be tried at the Crown Court.
[A] Wrong. A youth does not have the right to elect Crown Court trial.
[B] Wrong. The test for the magistrates’ court is whether it is in the interests of justice for Helen to be tried in the Crown Court with Susan. Given that there is very little difference in age, Helen is 17 years old (so at the top end of the youth age range), Helen is accused of playing the major role and has numerous previous convictions so it is likely that the court will consider it is in the interests of justice for Helen to be tried at the Crown Court with Susan
[C] Wrong. Given the stated preference for answer D, it is wrong to say that trial for Helen in the Crown Court is unlikely. The second part of C is also wrong in that, if her case was not going to the Crown Court, she could be tried in either the magistrates’ court or the youth court.
[D] Correct. See feedback for [B] above.

How well did you know this?
1
Not at all
2
3
4
5
Perfectly
12
Q

You represent Kevin, who attends the magistrates’ court on the day of his trial and has a conference with you. You go to speak to the prosecutor about the case and when you have finished you cannot find Kevin. Members of court security staff tell you that Kevin got into an argument with another court user and was very abusive, which resulted in Kevin being arrested and taken to the police station. Enquiries reveal that Kevin is not likely to be released before the end of the court day.
The magistrates are ready to hear Kevin’s case and the prosecution are ready for trial. You apply for the magistrates to adjourn the trial.
What are they most likely to do?
[A] Proceed to trial because the reason for Kevin’s failure to appear is not an acceptable one as he has misbehaved at court.
[B] Proceed to trial because there is a statutory presumption in favour of doing so, and the court is not required to enquire into the reasons for non-attendance.
[C] Adjourn the trial because it would not be in the interests of justice to proceed as Kevin’s absence is involuntary.
[D] Adjourn the trial because Kevin’s right to a fair trial includes a right to be present and effectively participate.

A

ANSWER
[C] Adjourn the trial because it would not be in the interests of justice to proceed as Kevin’s absence is involuntary.
[A] Not the best answer. This echoes s.11(2A) of the MCA 1980; ‘the court shall not proceed in the absence of the accused if it considers there is an acceptable reason for his failure to appear’. In applying this test, however, the court would be likely to find that there is an acceptable reason in that the absence is involuntary.
[B] Not the best answer. There is such a presumption in the case of an adult (MCA 1980 s.11(1)(b)). Although the court is ‘not obliged to investigate if no reason is offered’, the reasons given will be taken into account.
[C] The best answer, correctly reflecting the test in MCA s.11(1)(b) and the likely outcome in light of the ‘involuntary absence’ cases. See also the power to adjourn.
[D] Not the best answer as it does not sufficiently reflect the reasoning which is to be found in the MCA s.11 and the case law referenced above.

How well did you know this?
1
Not at all
2
3
4
5
Perfectly
13
Q

You defend Darren, who is charged with ABH. It is alleged that after a minor car accident with Jim, Darren jumped out of his car shouting, “Look what you’ve done, I’m going to kill you!” Darren then punched Jim in the head causing a cut to Jim’s head. Darren denies that he shouted at Jim. He alleges that Jim deliberately drove at him and started a fight and that he, Darren, was only acting in self-defence. At trial, the prosecution want to adduce evidence of Darren shouting “Look what you’ve done, I’m going to kill you!”
What is the best advice to give Darren about the admissibility of this evidence?
[A] It is evidence of bad character within the meaning of the CJA 2003, so it may be admissible because it is relevant to an important matter in issue between the defendant and the prosecution.
[B] It is not evidence of bad character within the meaning of the CJA 2003, because it has to do with the alleged facts of the offence, but it may be admissible as it is relevant evidence.
[C] It is not evidence of bad character within the meaning of the CJA 2003, because it has to do with the alleged facts of the offence, but it may not be admissible as the prosecution have not charged Darren with threats to kill.
[D] It is evidence of bad character within the meaning of the CJA 2003, so it may be admissible because Darren has made an attack on Jim’s character.

A

ANSWER
[B] It is not evidence of bad character within the meaning of the CJA 2003, because it has to do with the alleged facts of the offence, but it may be admissible as it is relevant evidence.
[A] Not the best answer. It is not evidence of bad character within the meaning of the CJA 2003, as it has to do with the alleged facts of the offence, but it will be admissible as it is relevant to the facts of the offence.
[B] The best answer. The CJA 2003, s.98(a) states that evidence which has to do with the alleged facts of the offence is not evidence of bad character. It is therefore excluded from the definition of s.98, and is admissible, subject to relevance, as evidence of misconduct “which has to do with the alleged facts of the offence charged”. See also McNeill (2008) 172 JP 50 where the evidence admitted was a statement made two days after an alleged offence of threats to kill, in which the D reiterated to a third party her threat to kill the same person. Here, the threat is made at the time of the offence and is to do with the facts of the offence and is relevant to rebut his claim that he acted in self-defence.
[C] Not the best answer. It is not evidence of bad character, however the prosecution does not need to charge the separate offence in order for this to be admissible. See s.98(a) CJA 2003 (see A above).
[D] Not the best answer. It is not evidence of bad character. See s.98(a) CJA 2003 (see A above).

How well did you know this?
1
Not at all
2
3
4
5
Perfectly
14
Q

You represent Michael at his PTPH in the Crown Court. Michael has pleaded guilty to an offence of failing to take reasonable general fire precautions to ensure that premises are safe, contrary to the Regulatory Reform (Fire Safety) Order 2005 (an either-way offence with a maximum sentence of two years’ imprisonment). Michael is a private landlord who owns several multiple occupancy houses and he has acquired significant personal wealth as a result of his business activities. An inspection by the local authority of one of Michael’s houses revealed that the fire alarm system was disconnected from the mains power supply and was therefore ineffective. Following Michael’s guilty plea, the judge ordered a pre-sentence report and adjourned the matter for sentence.
At the sentencing hearing, the judge determines that the offence crosses the custody threshold. The author of the pre-sentence report has commented that none of the usual community requirements would serve a useful purpose in Michael’s case.
What is the most appropriate sentence for you to commend to the judge when she sentences Michael?
[A] A short immediate custodial sentence.
[B] A suspended sentence with an unpaid work requirement.
[C] A suspended sentence and a fine.
[D] A suspended sentence and a conditional discharge.

A

ANSWER
[C] A suspended sentence and a fine.
[A] is not the best answer. The judge has found that the offence crosses the custody threshold, but the maximum sentence for this offence is 2 years, so a suspended sentence could be ordered. There is nothing in the fact pattern which would make the passing of a suspended sentence unlawful, and you would not mitigate for immediate custody if a more favourable outcome was possible – see C below.
[B] is not the best answer. Unpaid work can be combined with a suspended sentence, but this is not the most appropriate sentence for Michael as it is more onerous for him to complete unpaid work than to pay a fine. Also, the probation officer has made an observation that none of the usual community requirements would serve a useful purpose here, and so [C] is the more appropriate sentence for the reasons set out below.
[C] is the best answer. A fine may be combined with a suspended sentence. Further, such a sentence is particularly apt when the offending is related to a defendant’s business or employment, when dealing with offenders of substantial means. Both these features are present.
[D] is not the best answer. A suspended sentence cannot be combined with a discharge when sentencing for a single offence.

How well did you know this?
1
Not at all
2
3
4
5
Perfectly
15
Q

You represent Alfred, who is charged with robbery, taking a motor vehicle without the owner’s consent (a summary offence) and driving while disqualified (a summary offence). It is alleged that he robbed a woman of her handbag and then drove away in a stolen car. He will be pleading not guilty to all offences and wants to know if he can be tried in the magistrates’ court for some or all his offences.
What should you advise him?
[A] The magistrates are likely to send all the offences to the Crown Court for trial.
[B] The indictable offence will be sent for trial to the Crown Court and the summary offences will stay in the magistrates’ court for trial.
[C] The indictable offence will be sent for trial to the Crown Court and the summary offences will be sent to the Crown Court for plea only.
[D] The indictable offence will be sent for trial and the summary offences will be committed for sentence to the Crown Court.

A

ANSWER
[A] The magistrates are likely to send all the offences to the Crown Court for trial.
[A] This is the best answer. The summary offences are linked offences to which s.40 applies and it is in the interests of justice for them to be tried together.
[B] This is not the best answer. It is unlikely that the court will order find it is the interests of justice to separate the trials given that the offences are linked and s.40 applies.
[C] This is not the best answer. The summary offences are s.40 offences so, if they are sent to the Crown Court, they are sent for trial rather than plea.
[D] This is not the best answer. Summary offences cannot be committed for sentence.

How well did you know this?
1
Not at all
2
3
4
5
Perfectly
16
Q

You prosecute Robert, who is charged with ABH. The prosecution case is that Robert hit the complainant on the back of the head. The complainant states he did not see his attacker and has no idea who it was. Robert denies the offence. An eyewitness, Steve, made a statement containing an account of the assault which supports the prosecution case, in which he describes where he was standing, the distance between him and the incident and the quality of his view, together with a description of the attacker.
Steve gave his details to the police but said he did not want to give evidence because he was frightened of violent repercussions, as in his area anyone who helps the police is targeted by gangs. The police attempted a number of times to persuade him to give evidence and explained special measures directions to him. Steve then made a second witness statement saying that since the incident he has seen someone loitering outside his house and has had a brick thrown through his window. He states that he will not give evidence even if special measures are put in place. Steve has two previous convictions for shoplifting, dating back five years.
Two other witnesses, who are prepared to attend court, identify a man they saw running away from the scene as Robert, and their descriptions of Robert match Steve’s description of the attacker.
You apply to adduce Steve’s statement about the incident under CJA 2003 s.116(2)(e) on the ground of fear.
What decision is the Court most likely to make in relation to the application?
[A] Refuse to admit the evidence because the police have not made all reasonable efforts to get Steve to court.
[B] Refuse to admit the evidence because there is no evidence of intimidation by Robert and therefore the ground of fear cannot be established to the required standard.
[C] Refuse to admit the evidence because it is central to the case and Steve’s previous convictions put his reliability in doubt.
[D] Admit the evidence because it is not the sole evidence; the jury can take Steve’s background into account if the judge considers it relevant and there is other evidence against which his reliability can be measured.

A

ANSWER
[D] Admit the evidence because it is not the sole evidence; the jury can take Steve’s background into account if the Judge considers it relevant and there is other evidence against which his reliability can be measured.

[A] Not the best answer. A lack of effort to get the witness to court would hinder admissibility but it is not the only factor to consider, and here the police have made a number of attempts to persuade Steve to give evidence, including discussing the possibility of special measures.
[B] Not the best answer. Evidence of intimidation by Robert would strengthen the prosecution position but there is no requirement that fear is attributable to the accused. Showing the causal link between fear and failure to testify, and specific evidence of fear, are more important.
[C] Not the best answer. Steve’s evidence is important but there is no hard and fast rule making it thereby inadmissible and there are tools available, including other witnesses, for testing his reliability. Previous convictions would be available as if the witness were present: CJA s.124.
[D] The best answer. See [C]. Also, the evidence is capable of challenge on the basis of quality of view etc.

How well did you know this?
1
Not at all
2
3
4
5
Perfectly
17
Q

You represent Shane, who appears before the magistrates’ court charged with theft of ten bottles of champagne (value £190) and criminal damage (value £2,000). The circumstances are that while in a supermarket he approached a stand containing bottles of champagne bottles. He grabbed ten of them and put them into his holdall. He then pushed the stand over, smashing the rest of the champagne bottles, and left without paying. He wants to know whether he can have a trial in the Crown Court in respect of both charges.
What is the correct advice to give him?
[A] No, as each of the offences is summary only he cannot elect Crown Court trial.
[B] Yes, as he can elect Crown Court trial for the theft and the criminal damage is a related offence.
[C] No, as the criminal damage is a summary only offence, he cannot elect Crown Court trial for either of the offences.
[D] Yes, as each of the offences is triable either way, he can elect Crown Court trial.

A

ANSWER
[B] Yes, as he can elect Crown Court trial for the theft and the criminal damage is a related offence
[A] is wrong. The criminal damage is treated as triable only summarily because of its value. Although the theft is a low value shoplifting and therefore triable only summarily, s22A(2) still allows the right of election for the theft.
[B] is correct. The theft is a low value shoplifting and therefore triable only summarily, s22A(2) still allows the right of election for the theft. The criminal damage is a related offence and will also be sent to the Crown Court for trial under s51(3) CDA 1998.
[C] is wrong. He has no right of election for criminal damage because it is treated as a summary offence, but this does not stop his right of election for the theft.
[D] is wrong. The offence of criminal damage is not triable either way due to its value. Whilst the theft offence is low value shop theft, it retains its right of election by virtue of s22A(2) MCA 1980.

How well did you know this?
1
Not at all
2
3
4
5
Perfectly
18
Q

You are defending Gavin, who is being prosecuted for historic sex offences against ten children. At the trial, the prosecution call Debbie, a parent of one of the children, to testify about what her child had told her about Gavin. During cross-examination, in response to a question that you have asked, Debbie says, “It is obvious that he has committed the offences; ten children wouldn’t all be making it up.”
Is Debbie’s response admissible?
[A] It is admissible. Debbie may express an opinion from the facts she has personally perceived about whether Gavin has committed the offence.
[B] It is inadmissible. Debbie may only give evidence of facts she has personally perceived and not evidence of her opinion of Gavin.
[C] It is inadmissible. Debbie may not give evidence of her opinion because she is not an expert on the commission of sex offences.
[D] It is admissible. Debbie may express her opinion of whether Gavin has committed the offence because this is not a matter calling for expertise.

A

ANSWER:
[B] It is inadmissible. Debbie may only give evidence of facts she has personally perceived and not evidence of her opinion of Gavin.
[A] Wrong: the general rule is that witnesses may only give evidence of facts they personally perceived and not evidence of their opinion, i.e. evidence of inferences drawn from such facts. The opinion Debbie is giving is not a permitted exception to the rule for ‘non-experts’
[B] This is the correct answer. See the answer to [A].
[C] Wrong. Debbie has not been called to provide expert opinion evidence but to testify to what her child told her them about Gavin. What Debbie’s child said to Debbie is not a matter calling for expertise.
[D] Wrong. Debbie may not express her opinion in this situation. The opinion Debbie is giving is not a permitted exception to the rule for ‘non-experts’. What Debbie’s child said to Debbie is not a matter calling for expertise. It does not follow that witnesses are qualified to express an opinion in matters not calling for expert opinion.

How well did you know this?
1
Not at all
2
3
4
5
Perfectly
19
Q

You are the judge in the case of Danny and yesterday you sentenced him to two years’ imprisonment for ABH. Danny was not represented as he had recently sacked his legal team, and you were not informed that he was on the hospital wing of the prison prior to attending for the sentencing hearing. Today, you have been sent a psychiatric report which was commissioned by his former solicitors, which concludes that there is a significant risk that Danny will commit further serious violent offences. In light of this report, you wish to hear submissions and make a decision as to whether Danny is a dangerous offender within the meaning of the CJA 2003 s.226A.
What action should you take in order to vary Danny’s sentence?
[A] You should use your discretion to rescind the original sentence and re-sentence Danny without a hearing.
[B] You should re-list the case within 56 days in order to vary the sentence (under the “slip rule”).
[C] You should invite the Attorney General to refer the sentence to the Court of Appeal, as only the Court of Appeal can vary a sentence once it has been pronounced in open court.
[D] You should take no action; the power to vary sentence cannot be used to impose a more punitive sentence and it is for the prosecution to decide whether to refer the sentence to the Court of Appeal.

A

ANSWER
[B] You should re-list the case within 56 days in order to vary the sentence (under the “slip rule”).
[A] Wrong. A sentence must be pronounced, reasons given, and the effect of the sentence explained.
[B] Correct. A Crown Court judge can vary a sentence, including by imposing a more punitive sentence, within 56 days of the sentence being passed: Sentencing Act 2020 s.385
[C] Wrong. A Crown Court judge can vary a sentence, including by imposing a more punitive sentence, within 56 days of the sentence being passed: see [B] above.
[D] Wrong. The slip rule can be used to impose a more punitive sentence.

How well did you know this?
1
Not at all
2
3
4
5
Perfectly
20
Q

HHJ Singh has been trying a case at the Crown Court involving an allegation of robbery. Evidence has been given of the Defendant’s police interview, during which he advanced a defence of alibi. However, while giving evidence in the trial, the Defendant admitted lying to the police and now relies upon the defence of duress. The judge has been considering whether to give the jury a Lucas direction or a direction under s.34 of the Criminal Justice and Public Order Act 1994. After much consideration she has decided to give both.
Tomorrow, counsel will make their final speeches and the judge will sum up the case to the jury. The judge is concerned about what, if any, action she should take concerning the proposed Lucas / s.34 directions she is proposing to make.
Which of the following best describes what the judge should do in the circumstances?
[A] She should invite representations from counsel, immediately before she sums up the case to the jury, as to the appropriateness of the proposed directions.
[B] She should invite representations from counsel, immediately after the summing-up, but before the jury’s retirement, as to the appropriateness of the directions given.
[C] She should invite representations from counsel, before they make their final speeches to the jury, as to the appropriateness of the proposed directions.
[D] She can proceed to sum up the case without seeking representations from counsel because if she errs in her summing-up, the defence has the option of appealing any conviction to the Court of Appeal.

A

ANSWER
[C] She should invite representations from counsel, before they make their final speeches to the jury, as to the appropriateness of the proposed directions.
[C] best describes what the Judge should do. As the Judge has more than one option as to how to sum up the law, the best course would be to seek the representations of counsel. This should be done before speeches. It follows that the advice in [A] does not best describe what the Judge should do: counsel should know how the judge proposes to sum up the law before they make their speeches.
The course in [B] will only be appropriate in very exceptional circumstances: R v Cocks. No exceptional circumstances are indicated by the fact pattern.
Whilst in many cases it is not necessary to seek representations from counsel as to how the law should be dealt with, it has become increasingly common to do so where there is a possibility of misunderstanding or doubt. For that reason, the advice in [D] is not the best.

How well did you know this?
1
Not at all
2
3
4
5
Perfectly
21
Q

The defence wish to challenge the quality of the prosecution evidence in a case of s.20 GBH which is being sent for trial in the Crown Court.
What is the correct course of action for the defence?
[A] No challenge may be made until the end of the prosecution case at the defendant’s Crown Court trial.
[B] An application for dismissal may be made to the Crown Court.
[C] An application for dismissal must be made in the magistrates’ court.
[D] An application for dismissal may only be made in writing to the Crown Court.

A

ANSWER
[B] An application for dismissal may be made to the Crown Court.
[A] Wrong. CDA 1998 sch. 3 para.2 makes it clear that an application for dismissal may be made well before the actual trial of a Defendant.
[B] Correct. CDA 1998 sch. 3 para.2 makes it clear that such an application may be made.
[C] Wrong. An application for dismissal is made to the Crown Court: CDA 1998 sch. 3 para.2.
[D] Wrong. An application for dismissal is made to the Crown Court: CDA 1998 sch. 3 para.2. The application can be made orally (after giving written notice) or in writing.

How well did you know this?
1
Not at all
2
3
4
5
Perfectly
22
Q

André is being tried for rape. The prosecution case is that the complainant, Lisa, met André at a club and invited him to her flat. Lisa says André had sex with her without her consent while in the flat.
Lisa reported the matter to the police. She told them she knew his name and where he worked because he was wearing a T-shirt with the company name on. She also told the police that he spoke with a South African accent.
The officer in the case, DS Harper, called André at work and recorded the call. DS Harper introduced herself as a police officer and asked André to confirm his name and where he was, which he did. She then explained that a complaint had been made by Lisa and asked him questions about the incident. André told DS Harper that he had met Lisa and gone back to her flat where they had consensual sex. After the call, André was arrested and interviewed, during which he refused to answer any questions.
In André’s defence statement he denies the offence entirely and says he did not go back to Lisa’s flat. The defence apply to have the entire recording of the phone call excluded under PACE 1984 s.76 because DS Harper had not cautioned André before questioning him about the incident. The prosecution agrees that the questions and answers regarding the alleged offence should be excluded in this case. However, in order to demonstrate that André speaks with a South African accent, the prosecution want to play the beginning of the telephone call at trial, where André confirmed his name and where he was.
Which of the following is the trial judge most likely to do in response to the prosecution application?
[A] Allow the jury to listen to those parts of the recording which are necessary to demonstrate André’s accent, but should refuse to allow them to hear any part of it that is adverse to him.
[B] Refuse to allow the jury to listen to any part of the recording because the Prosecution agree that the questions and answers about the incident should be excluded under s.76.
[C] Allow the jury to listen to the recording because it is admissible to demonstrate André’s accent. She should then give the jury a direction about why the recording is being played and tell them not to take account of the confession when deciding the case.
[D] Make no ruling but indicate that she will revisit the issue at the close of the defence case. Provided that the Defendant has given evidence, so that the jury are able hear his accent, she will refuse to allow the recording to be played.

A

ANSWER
[A] Allow the jury to listen to those parts of the recording which are necessary to demonstrate André’s accent, but should refuse to allow them to hear any part of it that is adverse to him.
[A] is the best decision. Section 76(4)(b) makes admissible as much of the excluded confession evidence as is necessary to show that the accused speaks in a particular way. Care must be taken to avoid prejudice to the accused when adducing such evidence; s. 76(4)(b) permits the prosecution to adduce only ‘so much of the confession as is necessary to show’ the relevant feature, but even this may in some cases be impossible without the jury becoming aware that a confession has been made.” In this case the jury should be played those parts of the recording that allow them to judge André’s accent (i.e. his personal details and location, but not the portions of the conversation where he makes admissions about the offence).
[B] is not the best decision. This answer does not take account of section 76(4)(b) which specifically makes provision for a confession excluded under section 76 to be admitted (or such parts of it are necessary) in order to show the accused speaks in a particular way.
[C] is not the best decision. This course of action is not legally wrong and a direction to the jury could reduce the prejudice suffered by the accused in certain cases. Further, there may be situations where the whole confession has to be admitted in order to give effect to section 76(4)(b) but that is not the case here.
[D] is not the best decision. This answer appears to put pressure on the defendant to have to give evidence in his own trial, which he cannot be compelled to do. The question for the court in this case is whether the recording (or parts of it) can safely be admitted pursuant to section 76(4)(b), if it can then it should be admitted. If it cannot then the court can use section 78 or common law to exclude it. The answer is not contingent on whether D gives evidence irrespective of whether this would achieve the same end as playing the recording.

How well did you know this?
1
Not at all
2
3
4
5
Perfectly
23
Q

cross examining the principal prosecution witness, Fred (George’s employer), you put to Fred that he had invented the alleged loss in order to further an insurance claim. George is not going to give evidence. He has a previous conviction from 33 years ago for indecent (now known as ‘sexual’) assault on a young boy. The prosecution seek to adduce evidence of this conviction as bad character under CJA 2003 s.101 (1)(g).
What is the best argument with which to oppose the prosecution’s application?
[A] That the previous conviction does not amount to bad character because it is old and spent.
[B] That the prosecution should not be allowed to adduce his previous conviction because George is not giving evidence.
[C] That the court must not admit evidence of the previous conviction because it would have such an adverse effect on the fairness of proceedings that the court ought not to admit it.
[D] That the previous conviction is not admissible as it is not relevant to an important matter in issue between the defendant and the prosecution.

A

ANSWER
[C] That the court must not admit evidence of the previous conviction because it would have such an adverse effect on the fairness of proceedings that the court ought not to admit it.
[A] is not the best answer because the fact that the conviction is old does not mean that it cannot be bad character, but this would be part of the s.101(3) argument.
[B] is not the best answer because the court may admit evidence of bad character even where a defendant is not giving evidence.
[C] The prosecution is making the application under gateway s101(1)(g). Although you might argue that you had done no more than deny guilt, given the fact that you suggested a purpose for the telling of lies by the witness this is unlikely to succeed. Since the application is under s101(1)(g), s101(3) applies and an argument under that provision is the best one to make in the circumstances.
[D] is wrong because s.101(1)(d) propensity is not the appropriate gateway in this case.

How well did you know this?
1
Not at all
2
3
4
5
Perfectly
24
Q

You are defending Nellie at her forthcoming PTPH on one count of aggravated burglary (an indictable-only offence). The prosecution have served evidence of their case against Nellie, who denies participation in the offence.
The prosecution case is that the burglar knocked on the door of an elderly man, threatened him, tied him to a chair and stole a laptop and mobile phone. There is no evidence that the burglar had a weapon with her when she entered the property or at any other time during the commission of the offence (an essential ingredient of an aggravated burglary).
What is the best approach to take in respect of the aggravated burglary?
[A] At the PTPH, you should identify on the PTPH form the absence of evidence of a weapon as an issue for trial.
[B] At the PTPH, you should not make any reference to the absence of evidence of a weapon, but instead should raise it at trial in a submission of no case to answer.
[C] Prior to the PTPH, you should draft and serve a written notice applying to dismiss the charge.
[D] At the PTPH, after Nellie has entered her not guilty plea, you should make an oral application to dismiss the charge.

A

ANSWER
[C] Prior to the PTPH, you should draft and serve a written notice applying to dismiss the charge.
[C] Is the best approach because there is not sufficient evidence for Nellie to be properly convicted of the aggravated burglary and she is therefore entitled to apply for it to be dismissed. This must be done prior to the date of arraignment and can only be done if the defendant “gives written notice of intention to do so.”
[A] Is not the best approach because it does not allow for an application to dismiss to take place. The defendant’s best interests are served by having the aggravated burglary dismissed before she is required to enter a plea to it and stand trial. If it is not dismissed at this stage there is a chance that the prosecution could rectify the evidential defect prior to any trial.
[B] Is not the best approach for the same reason that is set out in [A] above.
[D] Is not the best approach because once Nellie has been arraigned the court has no power to entertain an application to dismiss a charge or count. Additionally, an oral application may not be made unless written notice has been given.

How well did you know this?
1
Not at all
2
3
4
5
Perfectly
25
Q

You represent Jack, who has pleaded guilty to an offence of aggravated vehicle taking (an either-way offence). Jack admits to using a screwdriver to break into his father’s car, which he then took without consent. Whilst Jack was driving the car he was involved in a road traffic accident which resulted in damage to the car. Jack was found in the driver’s seat and was arrested. When he was booked into custody and searched, £100 in cash and a screwdriver were found in his jacket pocket.
Jack wants to know whether the magistrates can order deprivation of the car, screwdriver and cash.
What is the correct advice to give Jack?
[A] They should not order deprivation of the car, as it belongs to his father, but they could require Jack to forfeit the screwdriver and the cash.
[B] They could require Jack to be deprived of the car and the screwdriver as both are items which were used to commit or facilitate the offence, but not the cash as it was not used to facilitate the offence.
[C] They should not order deprivation of either the car or the cash, because the car belongs to his father and the cash was not used for the purpose of committing the offence, but they could require Jack to forfeit the screwdriver.
[D] They could require Jack to be deprived of the car, the screwdriver and the cash as they were all in his possession at the time of arrest.

A

ANSWER
[C] They should not order deprivation of either the car or the cash, because the car belongs to his father and the cash was not used for the purpose of committing the offence, but they could require Jack to forfeit the screwdriver.
[A] Wrong, as following S.A. 2020 s.153(3) any item for deprivation needs to have been used for the purpose of committing or facilitating the offence. The cash was not so used. Also, the power should not be extended to items which are not owned by the defendant. See Hall [2015] RTR 86(9), 2014 EWCA Crim 2413.
[B] Wrong as the power of deprivation should not be extended to items which are not owned by the defendant.
[C] Correct. The order does not fall foul of either of the points raised in [A] or [B] above.
[D] Wrong. Deprivation of the car under s.153 is not possible as it does not belong to Jack, and the cash was not used for the purpose of the offence (see [A] above).

How well did you know this?
1
Not at all
2
3
4
5
Perfectly
26
Q

You are defending Rahim, who has been charged with s.18 wounding. The allegation against Rahim is that he was part of a violent gang outside a football stadium where he struck PC Evans with a broken bottle and then fled the scene. PC Evans later gave a statement in which he described a man coming out of the crowd and striking him with a bottle. PC Evans stated that although it was night-time, the area was well lit, and he had a good view of his attacker for about 5 seconds.
Two months later, Rahim was arrested after PC Evans saw him in the street and recognised him as his attacker. When interviewed under caution, Rahim denied being present outside the stadium and claimed that he was at his brother’s house at the relevant time.
At trial, Rahim was cross-examined on the basis that he had fabricated his alibi; that he was at the scene of the disorder, and he was responsible for the attack on PC Evans. Rahim denied this. His brother Micah initially supported Rahim’s alibi. When Micah gave evidence, however, the prosecution was able to demonstrate from social media posts that on the night of the incident, Micah was abroad. Micah accepted he must have been mistaken and that he was in fact with his brother the following evening after his return to the UK.
At the end of the trial, the judge has indicated that he is going to give a Turnbull direction. The prosecution argue that the jury are entitled to use the false alibi as support for PC Evans’ identification of Rahim at the scene. You want to argue that the jury should not be able to use the evidence to support the identification.
What is your best submission?
They may only use the false alibi as supporting evidence if:
[A] they are sure that Rahim’s brother, Micah, deliberately made up the alibi and can discount any possible innocent motive for it.
[B] they are sure that Rahim deliberately made up the alibi and can discount any possible innocent motive for it.
[C] they are sure that the alibi is untrue.
[D] they are first sure that PC Evans has correctly identified Rahim as his attacker.

A

ANSWER
[B] they are sure that Rahim deliberately made up the alibi and can discount any possible innocent motive for it.
[A] is not the best submission. The crucial question for the jury is whether by providing a false alibi or telling lies the defendant has incriminated himself and consequently whether that self-incrimination can support the identification evidence. The alibi was initially given by Rahim and although Micah has resiled from it, the jury could not use that as supporting evidence unless they were sure that Micah had deliberately lied about being with his brother and that there was no innocent explanation for the lie. The third condition set out in relation to proving the lie is already met by the social media evidence.
[B] is the best submission. The governing principles in relation to self-incrimination by false alibis or other lies, as set out by the Court of Appeal in Lucas [1981] QB 720, have now been held applicable in identification cases. Before such lies can be regarded as supporting an identification, they must accordingly be shown to be deliberate and material; the court or jury must be able to discount any possible innocent motive for the lies and they must be proved to be lies by evidence other than the identification(s) that they are to support. Consequently, this submission represents the best restatement of the applicable law.
[C] is not the best submission. The submission is not wrong, but it does not go far enough. The jury must be sure that the alibi is false, but they also have to be sure that Rahim deliberately invented it and they must be able to discount the possibility of an innocent motive for the false alibi.
[D] is not the best submission. The point here is whether the self-incriminating evidence of the false alibi (if it is false) is capable of supporting PC Evans’ identification evidence. If the jury were sure that PC Evans’s identification was correct, then there would be no need to seek support for their finding in the false alibi.

How well did you know this?
1
Not at all
2
3
4
5
Perfectly
27
Q

You prosecute Seamus, who is charged with raping Stephanie. Seamus denies the offence. He claims that they kissed but did not have sexual intercourse. The case has been sent to the Crown Court for trial.
During the investigation, the police downloaded several hundred text messages passing between Stephanie and her best friend Carol, many of which refer to Seamus. These messages do not appear either to undermine the prosecution case, nor do they appear to assist the defence case.
You are asked to advise as to the obligations of the police and of the Crown Prosecution Service in relation to these text messages.
Which of the following statements is correct?
[A] The police have an obligation to retain a record of the text messages if they could reasonably be considered capable of undermining the case for the prosecution or assisting the case for the defence.
[B] The police have an obligation to retain a record of the text messages if they may be relevant to the investigation, whether or not they appear to assist either party.
[C] The disclosure officer should include the record of the text messages as an item on a schedule of sensitive unused material.
[D] The Crown Prosecution Service should disclose the record of the text messages to the defence as part of initial disclosure.

A

ANSWER
[B] The police have an obligation to retain a record of the text messages if they may be relevant to the investigation, whether or not they appear to assist either party.
[A] is wrong because material does not have to fall into CPIA s.3 in order for there to be an obligation to be retained. The correct test is as set out in [B] below.
[B] is correct as investigators are responsible for recording and retaining all material which ‘may be relevant to the investigation’.
[C] is wrong. The only material that should be included on the sensitive material schedule is material that would give rise to a real risk of serious prejudice to an important public interest if disclosed.
[D] is wrong as the material does not pass the test in CPIA s.3.

How well did you know this?
1
Not at all
2
3
4
5
Perfectly
28
Q

Forest (aged 14) wishes to appeal against his conviction and sentence. He appeals to the Crown Court.
Which of the following statements is correct?
[A] The composition of the panel to hear the appeal at the Crown Court must always comprise a circuit judge or recorder and two lay magistrates (one man and one woman).
[B] The defence must give notice of appeal in writing within 20 business days of the sentence being passed.
[C] An appeal against conviction is a complete rehearing of the trial. If the parties wish to call evidence which was not relied upon at trial in the lower court, they must apply for leave from the Court to rely upon any new evidence.
[D] If notice of appeal is served within the time limit, no leave to appeal is required.

A

ANSWER
[D] If notice of appeal is served within the time limit, no leave to appeal is required.
[A] Is wrong because in exceptional circumstances the Crown Court may include only one Justice of the Peace and need not include both a man and a woman if the presiding judge decides that the hearing will otherwise be reasonably delayed.
[B] Is a wrong answer - because notice of appeal must be given in writing within 15 business days of sentence being passed.
[C] Is a wrong answer - because leave is not required to call new evidence.
[D] Is the correct answer because - leave is not required if the notice is served within the relevant time.

How well did you know this?
1
Not at all
2
3
4
5
Perfectly
29
Q

Alicia is arrested for possession with intent to supply Class A drugs. She was interviewed by the police under caution, with a solicitor present, and made no comment to all questions asked. At her Crown Court trial, when giving evidence, Alicia states that the reason she made no comment was because her solicitor advised her to do so.
Which of the following correctly explains the effect of this statement?
[A] Alicia has waived privilege by revealing the advice from her solicitor.
[B] Alicia has not waived privilege, as this statement does not amount to a waiver of privilege.
[C] Alicia has not waived privilege as she had to provide such information in order to defend herself adequately.
[D] Alicia has not waived privilege because only her solicitor, as the person who gave the advice, can waive privilege.

A

ANSWER
[B] Alicia has not waived privilege, as this statement does not amount to a waiver of privilege.
[A] is wrong as by simply stating that she had received legal advice to answer no comment does not amount to a waiver of privilege – see Condron (1997).
[B] is correct. The reason given by an accused for not answering is that he has been so advised by his solicitor. That advice does not amount to a waiver of privilege.
[C] is wrong. Although Alicia has not waived privilege in these circumstances the reason given is wrong – just because a defendant needs to put something forward as part of their defence, does not protect them from waiving privilege. A defendant can waive privilege by answers they give while giving evidence.
[D] is wrong. Although Alicia has not waived privilege the reason is wrong. A waiver of privilege can occur whether the revelation is made by the accused or the solicitor on behalf of the accused.

How well did you know this?
1
Not at all
2
3
4
5
Perfectly
30
Q

You are prosecuting Tristan, who is charged with common assault at a party. Tristan denies being the aggressor. His brother Matthew, who was at the party, is due to give evidence in support of Tristan.
You call Alice, who was also at the party, to give evidence, and she describes Tristan as the aggressor. Once she has finished her evidence, you are informed by the officer in the case that, during Alice’s evidence, someone was in the public gallery making notes. The officer then saw that person talking to Matthew and the officer believes they were describing Alice’s evidence to him.
When Matthew gives evidence, you cross-examine him about whether he spoke to the person at court, which he denies.
What is the most appropriate course of action for you to take?
[A] Apply to exclude Matthew’s evidence as he has potentially received information from this person, which he should not have.
[B] As you have put the point to Matthew, you can comment about it in your closing speech, without needing to call further evidence on it.
[C] Call evidence to contradict Matthew’s denial.
[D] There are no further steps you should take, as you cannot call evidence to contradict Matthew’s denial.

A

ANSWER
[C] Call evidence to contradict Matthew’s denial.
[A] Is not the best answer as you should at this stage call evidence in rebuttal to contradict the account given by Matthew. Evidence to contradict a witness’s denial of bias or partiality is admissible, to show he is prejudicial concerning the case being tried (Mendy (1976) 64 Cr App R 4).
[B] Is not the best answer as you should at this stage call evidence in rebuttal (see above). Whilst you can comment in your speech, you should also call evidence in rebuttal of what has been seen in order to disprove Matthew’s account.
[C] Is the best answer. You should call evidence in rebuttal to contradict the account given by Matthew as evidence to contradict a witness’s denial of bias or partiality is admissible, to show he is prejudicial concerning the case being tried. See case of Mendy (1976) 64 Cr App R 4. In that case, the Court of Appeal held that the trial Judge had been right to allow the prosecution to call evidence in rebuttal as the husband had been involved in a situation which was designed to defeat the purpose of keeping prospective witnesses out of court to enable him to give more convincing evidence of his account.
[D] Is not the best answer as you should now call evidence in rebuttal of Matthew’s denial and are able to do so. See [C] above.

How well did you know this?
1
Not at all
2
3
4
5
Perfectly
31
Q

Mo and Simon were jointly charged with arson with intent to endanger life (an indictable offence) and the case was sent to the Crown Court. The allegation is that they set fire to the houses of two of their neighbours. In the time since Mo and Simon were sent to the Crown Court, both neighbours have died as a direct result of the fires. A joint count of murder has therefore been added to the draft indictment.
You are prosecution counsel and have been asked by the reviewing lawyer to advise when the draft indictment should be served on the Crown Court. What is the best advice to give?
[A] A draft indictment was automatically generated when the case was sent for trial and need not be served on the Court.
[B] It must be served not more than 28 days after the service of the prosecution case.
[C] It must be served more quickly than the normal 28-day period and must be served at least seven days before the PTPH.
[D] It must be served within 28 days of the case being sent to the Crown Court.

A

ANSWER
[C] It must be served more quickly than the normal 28-day period and must be served at least seven days before the PTPH.
[A] Though an indictment is typically automatically generated when the case is sent to the Crown Court, this is based on the allegations before the magistrates’ court and is ‘subject to substitution or amendment of the charges included by the prosecution’. The rule in Crim PR 10.3 requires that a prosecutor who substitutes or adds a count must serve that count on the Crown Court officer and the defendant. The facts here clearly give rise to additional charges/counts of either manslaughter or murder.
[B] This answer is correct, in as much as 28 days from the service of the prosecution case is the normal timescale (Crim PR 10.4). However, it fails to take into account the point explained under C, below.
[C] This is the best answer because CrimPD 5.2 requires the indictment to be served at least seven days before the PTPH, which itself takes place within 28 days of the case being sent to the Crown Court (CrimPD 6.3.8).
[D] This is not the best answer because it is actually 21 days from the case being sent to the Crown Court since the indictment must be served at least 7 days before the PTPH, which takes place within 28 days of the case being sent. 28 days would therefore be too late.

32
Q

At their joint trial, Mo and Simon both gave evidence and blamed each other for the offences. They were both found guilty of both of the offences charged. After his conviction, Mo wishes to apply for permission to appeal on the ground that he should have been tried separately from Simon.
What is the most likely outcome and rationale?
[A] Permission to appeal will be granted. The judge was obliged to order separate trials in this case because the risk of prejudice is unusually great where each defendant accuses and incriminates the other.
[B] Permission to appeal will be denied. The charges against the accused were not too many or too complicated for the jury to understand or cause prejudice to either defendant.
[C] Permission to appeal will be granted. The defendant was prejudiced or embarrassed in his defence by being charged with more than one offence. The counts against him should have been tried separately.
[D] Permission to appeal will be denied. It is at the judge’s discretion whether to separate the trial and, where the accused are jointly charged, the presumption in favour of a joint trial is very strong.

A

ANSWER
[D] Permission to appeal will be denied. It is at the Judge’s discretion whether to separate the trial and, where the Accused are jointly charged, the presumption in favour of a joint trial is very strong.
[A] Permission to appeal would be unlikely. Although this answer refers back to the cut-throat defence and the facts of the scenario, this rationale for why separate trials of the accused should have been ordered has been held not to be a convincing reason for severance.
Even if the accused are expected to blame each other for the offence (i.e. will run ‘cut-throat’ defences), the interests of the prosecution and the public in a single trial will generally outweigh the interests of the defence in not having to call each accused before the same jury to give evidence for himself which will incriminate the other (Grondkowski [1946] KB 369; Moghal (1977) 65 Cr App R 56; Edwards [1998] Crim LR 756; Crawford [1997] 1 WLR 1329).
See also Josephs (1977) 65 Cr App R 253:
‘… the fact that some of [a co-accused’s] statements may rub off on the other accused … is just one of those things that happens in the course of a multiple criminal trial. The advantages of having co-defendants tried together is so great that the right to order a separate trial will not be granted unless there is good reason for it.’
[B] It is likely that permission would be refused, but the rationale given here, although correct, is only a consideration and not as strong as the rationale in [D], nor does it state the importance of the discretionary nature of the power.
[C] Permission to appeal would be unlikely and the rationale, although the correct test for the severance of counts, does not mirror the ground of appeal – that he should have been tried separately to Simon. He is not applying for permission to appeal on the ground that his charges of manslaughter/murder and/or arson should have been tried separately. This muddles the law on severance of counts with the discretion to order separate trials of the accused.
[D] This is the best answer. Permission to appeal is likely to be denied and the rationale refers to the discretion to order separate trials and the presumption in favour of a joint trial where the accused are jointly charged.

33
Q

Dustin was convicted of burglary in the magistrates’ court and sentenced to a 12 month community order with an unpaid work requirement of 100 hours. Since the community order was imposed, Dustin has failed to attend the unpaid work requirement on four separate occasions, without reasonable excuse. He has now been summonsed to attend the magistrates’ court for breaching the order. Dustin admits the breach.
What option do the magistrates have regarding Dustin’s breach of the community order?
[A] They may impose a fine on Dustin for the breach.
[B] Their only option is to revoke the order and re-sentence Dustin for the original offence.
[C] They may allow the order to continue without taking any further action.
[D] They may commit Dustin to the Crown Court for sentence for the breach.

A

ANSWER
[A] They may impose a fine on Dustin for the breach.
[A] is correct – the magistrates can impose a fine not exceeding £2,500 (SA 2020 Sch. 10 para. 10)
[B] is wrong – this is an option but not the only option.
[C] is wrong – the magistrates must deal with the offender in one of the ways set out in para 9(1).
[D] is wrong – the order was imposed by the magistrates’ court so they have no such power.

34
Q

The youth court has made a finding of guilt against your client, Alana (aged 14), for ABH. It is her first offence. She seeks your advice on whether she will be made subject to a detention and training order.
What is the correct advice to give Alana?
[A] Given Alana’s age, the court has an unfettered discretion to pass a detention and training order.
[B] The court may pass a detention and training order if it is of the opinion that the offence is so serious that neither a fine alone nor a community sentence can be justified for the offence.
[C] The court has no power to pass a detention and training order because Alana is not a persistent offender.
[D] The court may pass a detention and training order if it is of the opinion that only a custodial sentence will be adequate to protect the public from further offences by Alana.

A

ANSWER
[C] The court has no power to pass a detention and training order because Alana is not a persistent offender.
[A] is wrong because despite being old enough to be eligible for a DTO the other condition (persistent offender) is not present.
[B] is wrong. It sets out the ‘custody threshold test’ but that is not the applicable test for making a DTO because Alana is not a persistent offender.
[C] is correct because Alana is under 15 so the court must be of the opinion that she is a persistent offender before making a DTO. She is not a persistent offender as she this is her first conviction .
[D] is wrong: the correct test for whether someone of 14 should receive a DTO is whether they are a persistent offender, as set out under [C].

35
Q

Sasha (aged 16) is making her first appearance in the youth court on a charge of s.18 GBH.
Sasha is currently a looked after child and is living in care home accommodation provided by the local authority. While at the home, she is alleged to have attacked another resident with a set of heated hair straighteners. This unprovoked and sustained attack caused serious burns to the face and arms of the 12-year-old victim. Sasha has been diagnosed with a learning disability although she was deemed fit to plead during recent, unrelated, criminal proceedings against her.
Before Sasha’s first appearance, she gives full instructions to her solicitor that she wants to admit the offence.
When the case is called on, Sasha becomes very agitated and begins to shout and swear at the magistrates. Despite being asked to calm down several times by the bench, her solicitor and her social worker, Sasha continues to be abusive and refuses to confirm her identity to the court.
What is the most appropriate course of action for the bench to take in these circumstances?
[A] Adjourn the proceedings to another day to enable Sasha to take part and enter a plea.
[B] Treat Sasha’s refusal to participate in the proceedings as a not guilty plea and go on to determine whether the case should be heard in the youth court or the Crown Court.
[C] Adjourn the proceedings and obtain a medical report to determine whether Sasha is fit to plead given her learning disability and behaviour in court.
[D] Exclude Sasha from the hearing and proceed in her absence by inviting her solicitor to enter a plea on her behalf.

A

ANSWER
[D] Exclude Sasha from the hearing and proceed in her absence by inviting her solicitor to enter a plea on her behalf.
[A] is not the best course of action. Although this course of action would not be wrong, it ignores the power under section 24B MCA 1980 to proceed in the absence of a defendant whose disorderly conduct before the court renders it impracticable to conduct the hearing in their presence. The accused is legally represented and all other requirements under the section are met in this case.
[B] is not the best course of action. Whilst a refusal to indicate a plea would be treated as a not guilty plea, this is a case where the defendant’s disorderly conduct is making it impossible to even begin the hearing let alone take a plea. Consequently, this is a case where the defendant’s disorderly conduct requires the hearing to take place in her absence.
[C] is not the best course of action. Sasha’s solicitor has been able to obtain full instructions on plea. She has, during previous proceedings against her, been deemed fit to plead and there is nothing to suggest that her behaviour before the court is indicative of an inability to understand the proceedings.
[D] is the best course of action. The plea before venue procedure can be determined in the absence of a young person who is legally represented, and the court considers that, because of the accused’s disorderly conduct before the court, it is not practicable for proceedings to be conducted in his or her presence, and the court considers that it should proceed in the absence of the accused. In such cases, the legal representative is invited to enter a plea on behalf of the accused.

36
Q

[A] It is on the prosecution and the standard is beyond reasonable doubt.
[B] It is on the defence and the standard is beyond reasonable doubt.
[C] It is on the defence and the standard is the balance of probabilities.
[D] It is on the prosecution and the standard is the balance of probabilities.

A

ANSWER
[C] It is on the defence and the standard is the balance of probabilities.
[C] is correct because the defence of insanity is an express statutory exception to the usual rule that the burden of proof is borne by the prosecution to a standard of beyond reasonable doubt. Where the legal burden is on the defence the standard is the balance of probabilities.
Therefore [A], [B] and [D] are wrong.

37
Q

Clive is being prosecuted for ABH on his stepson, Tom (aged 14). The prosecution allege that Clive hit Tom repeatedly across the face causing extensive bruising. Clive denies the offence and says Tom’s injuries must have been caused by someone or something else. Two years ago, Clive was cautioned for common assault after smacking Tom. The prosecution wish to introduce the caution as bad character evidence at trial.
Clive accepts he smacked Tom once, on the legs, when Tom was 12, but that it was a ‘one-off’ and he (Clive) learned his lesson and as a result would never have hit Tom across the face. Clive says that he wants to give evidence to this effect at his trial.
What is the most likely route of admissibility for Clive’s caution?
[A] The prosecution will seek to introduce evidence of Clive’s previous caution under gateway (d) to show that Clive has admitted striking Tom before and therefore is more likely to have committed the offence.
[B] The defence and prosecution will admit the evidence of Clive’s previous caution by agreement under gateway (a) because the prosecution will say it shows propensity while the defence will use it to explain that it was a ‘one-off’ from which Clive learned his lesson.
[C] The prosecution will introduce the evidence of Clive’s previous caution without it having to go through one of the gateways, because Clive’s caution is not bad character evidence and it is relevant to the issues in the case.
[D] The prosecution will not rely on the previous caution but the defence will introduce it themselves under gateway (b) to distinguish between the ‘one-off’ smack on the legs and the hits across the face.

A

ANSWER
[B] The defence and prosecution will admit the evidence of Clive’s previous caution by agreement under gateway (a) because the prosecution will say it shows propensity while the defence will use it to explain that it was a ‘one-off’ from which Clive learned his lesson.
[A] This is a good answer because the prosecution is likely to rely on Clive’s previous caution for smacking Tom: it is the same category of offence with which he is charged and is in respect of the same child. Therefore, gateway (d) is likely to apply but this is not the best answer because the defence also rely on the evidence of the caution – see the answer to [B].
[B] This is the best answer because, on the facts, both parties are likely to rely on the bad character evidence to support their case, as in the case of Kalu [2007] EWCA Crim 22.
[C] This answer is wrong because a caution is bad character evidence. Although a caution is not a conviction it still qualifies as misconduct or reprehensible behaviour. Therefore, it can only be admitted through one of the gateways.
[D] This is not the best answer because the prosecution will also want to rely on the caution. A more common reason for the defence to admit his own bad character through gateway (b) is to argue that he has never previously been convicted of an offence similar to that charged, which is not the case here.

38
Q

If the evidence of Clive’s caution is adduced at the trial, what is the direction that the judge is most likely to give to the jury about it?
The judge will
[A] give a bad character direction explaining its relevance and the reason the caution was admitted but will warn against giving it too much weight.
[B] treat Clive as having effective good character and will give a modified good character direction that takes account of the previous caution.
[C] give a bad character direction and will explain to the jury that because Clive has a caution, the jury may think he is a less credible witness than Tom.
[D] treat Clive as having absolute good character and give a full good character direction because a caution is not a conviction.

A

ANSWER
[A] give a bad character direction explaining its relevance and the reason the caution was admitted but will warn against giving it too much weight.
[A] This is the best answer. A bad character direction will be necessary because Clive’s caution has been adduced during the trial and, in accordance with Edwards [2005] EWCA Crim 1813, Hanson [2005] EWCA 824 and Sullivan [2015] EWCA Crim 1565, the jury should be told its relevance, why it has been admitted and warned against attaching too much weight to it.
[B] This is a good answer but it is not the best. Cautions, even for matters of a minor nature, cannot simply be ignored in the calculation of effective good character, relying on Martin [2000] 2 Cr App R 42. Under Hunter, Clive is unlikely to be treated as being of effective good character or receive a modified good character direction (in the judge’s discretion) because the caution was admitted during the course of the trial, it is relevant to the index offence and is the same type of offence against the same child. The previous caution is neither old nor irrelevant and this is not a situation where bad character was admitted under gateway (b) to differentiate the offence Clive is charged with from his previous antecedents. An accused who is not judged to be of effective good character and who has convictions or cautions is still entitled to adduce them under the CJA 2003, s. 101(1)(b) where they are relevant, for example to pave the way for an argument that he has no record for offences of the type charged. Hunter [2015] EWCA Crim 631, [2015] 1 WLR 5367 specifies that a modified good character direction may, in the judge’s discretion, be given regarding such an accused.”
[C] This is not the best answer. Although the judge is likely to give a bad character direction, the caution is unlikely to have been admitted for its relevance to an issue of credibility. Candidates who pick this answer may not understand the relevance of the caution to the issues in the case and muddle directions on bad character with good character (i.e. the two limbs of propensity and credibility).
[D] This is false. Clive cannot be regarded as having absolute good character under the approach in Hunter because he has a previous caution. A caution is capable of amounting to bad character.

39
Q

You are defending Liz, who is on trial in the Crown Court for fraud committed in the course of her employment. On the morning of trial, your instructing solicitor informs you that Liz’s colleague, Richard, sent an email to Liz the night before saying that Kasia, an important defence witness, has returned to Poland and no longer wishes to be involved in the proceedings.
Your instructing solicitor had previously been in regular contact with Kasia and has obtained a signed witness statement from her. However, the emails they have sent since yesterday were not delivered as the email address has been closed down and Kasia’s telephone number is now disconnected. Your instructing solicitor has no forwarding address. The CPS have also confirmed, on information from the police, that Kasia’s flat is empty.
What is the best course of action to ensure Kasia’s witness statement is admitted into evidence?
Submit that the evidence is admissible pursuant to
[A] CJA 2003 s.117
[B] CJA 2003 s.114(1)(d)
[C] CJA 2003 s.116 as Kasia cannot be found and reasonable steps have been taken to find her.
[D] CJA 2003 s.116 as Kasia is outside the UK and it is not reasonably practicable to secure her attendance.

A

ANSWER

[C] CJA 2003 s116 as Kasia cannot be found and reasonable steps have been taken to find her.

[C] Is the best answer given the circumstances of the case and the efforts that have been made by your instructing solicitor to keep track of Kasia up to the trial date and the steps that have been taken to identify where she now is.
[A] Is not the best answer because the more appropriate gateway is s116 rather than seeking to admit Kasia’s signed statement as a business document under s.117, given that you would also have to satisfy s.117(4).
[B] Is not the best answer given the circumstances of the case - s116 is a much more appropriate gateway to admissibility in the circumstances.
[D] Is not the best answer given the circumstances of the case, the efforts that have been made by your instructing solicitor to keep track of Kasia up to the trial date and the steps that have been taken to identify where she now is – which remains unknown. Although she told her colleague, Richard, that she had returned to Poland there is no direct evidence to support that. Therefore [C] is a stronger argument.

40
Q

Jamie (aged 43) has been convicted by a jury of possession with intent to supply a Class A drug. Jamie has two previous convictions for supplying Class A drugs, one of which was in 2017 and the other in 2019. All offences occurred in the UK.
What is the minimum sentence the judge will impose on Jamie?
[A] A mandatory minimum sentence of three years’ imprisonment.
[B] A sentence of at least three years’ imprisonment, unless there are particular circumstances which would make it unjust to do so in all the circumstances.
[C] A mandatory minimum sentence of seven years’ imprisonment.
[D] A sentence of at least seven years’ imprisonment, unless there are particular circumstances which would make it unjust to do so in all the circumstances.

A

ANSWER
[D] A sentence of at least seven years’ imprisonment, unless there are particular circumstances which would make it unjust to do so in all the circumstances.
[A] Wrong. The minimum sentence for a third Class A drug trafficking offence is at least seven years’ imprisonment (three years is the minimum for a third dwelling house burglary. Section 314(1), (2), (3)(a)-(b) of the Sentencing Act 2020.
[B] Wrong. The minimum sentence for a third Class A drug trafficking offence is at least seven years’ imprisonment. Sentencing Act 2020 s.313(1) & (2) only. The minimum sentence for a third domestic burglary is three years’ imprisonment.
[C] Wrong. The minimum sentence for a third Class A drug trafficking offence is at least seven years’ imprisonment but the judge need not impose this sentence if particular circumstances relating to any of the offences or to the offender would make it unjust to do so. Sentencing Act 2020 s.313(1) & (2) only.
[D] Correct. The minimum sentence for a third Class A drug trafficking offence is at least seven years’ imprisonment but the judge need not impose this sentence if particular circumstances relating to any of the offences or to the offender would make it unjust to do so.

41
Q

At around 23:30, Afsana saw her neighbour, Malcolm, assaulting his girlfriend with a knife. Afsana called the police. Minutes later PC Allam, who was on mobile patrol, arrived at the scene together with other officers. On arrival, he saw a male running away from the direction of Malcolm’s house. PC Allam stopped the male and asked if he was Malcolm, to which the response was “Yes”. At this point, PC Allam arrested and cautioned Malcolm at 00:00 hours. PC Allam then said, “Where’s your girlfriend?” Malcolm replied, “She’s in the front garden.”
Malcolm was taken to the police station where he was interviewed under caution by PC Allam. Before the interview, Malcolm told PC Allam that he had been drinking wine earlier that evening, but that he understood what was happening and the significance of the questions and his answers. During the interview, Malcolm asked, “What will happen if I refuse to respond to your questions?” PC Allam replied, “I am not allowed to answer that.”
Which of the following statements about PC Allam’s compliance with PACE 1984 is correct?
[A] He should have cautioned Malcolm before asking him to confirm his identity.
[B] He was entitled to question Malcolm as to the whereabouts of his girlfriend before taking him to the police station.
[C] He should not have conducted the interview until the following day due to Malcolm’s consumption of alcohol.
[D] He was correct in saying that he was not allowed to answer Malcolm’s question.

A

ANSWER
[B] Yes, he was entitled to question Malcolm as to the whereabouts of his girlfriend before taking him to the police station.
[A] is wrong: it is not necessary to caution someone where the question is solely to establish the person’s identity.
[B] is correct. Whilst normally, following a decision to arrest a suspect, he must be interviewed only at a police station, there is an exception where the delay would be likely (as here) to lead to physical harm to another person. It was clearly necessary, as soon as possible, to establish the whereabouts of Malcolm’s girlfriend.
[C] is wrong. A person can be interviewed, even if under the influence of alcohol, provided they are able to appreciate the significance of the questions and their answers.
[D] is wrong because if a suspect asks directly what action will be taken if he refuses to answer questions, the officer may answer.

42
Q

You are defending Kelly, who is being tried in the Crown Court for fraud. She denies the offence. The prosecution case includes an admission of guilt made by Kelly to the police while being interviewed under caution. The prosecution also seek to adduce a written letter of confession which was found in Kelly’s police cell after she told the police it was there during her interview. Kelly tells you that she did admit to committing the offence, but only because the police refused to give her any food or drink until she admitted her guilt. She also explains that she wrote the letter for the same reasons. Kelly was not legally represented during the interview. You successfully apply under PACE 1984 s.76 for the police interview to be excluded.
What is the best course of action for you to take in relation to the letter written by Kelly?
[A] Apply for the evidence to be excluded under PACE 1984 s.76.
[B] Apply for the evidence to be excluded under PACE 1984 s.78.
[C] Obtain prosecution counsel’s consent for Kelly’s account of how the letter came into existence to be admitted in evidence as an agreed fact.
[D] Use Kelly’s account of how the letter came into existence to undermine the credibility of the police officers during cross-examination.

A

ANSWER
[A] Apply for the evidence to be excluded under PACE s.76.
[A] The best answer. Kelly made the written confession, as she did the oral confession, in circumstances that are likely to render the confession unreliable.
[B] Not the best answer. Kelly made the written confession in circumstances that are likely to render the confession unreliable. Therefore, the strongest application would be to seek to exclude the evidence under s.76 of PACE. An application under s.78 would not be the principal argument in such circumstances.
[C] Not the best answer. Kelly made the written confession in what appear to be oppressive circumstances. Therefore, the principal response should be to seek to exclude the evidence under s.76 of PACE. Whilst theoretically possible it is highly unlikely that the prosecutor would agree to such an admission at this stage.
[D] Not the best answer. Kelly made the written confession in what appear to be oppressive circumstances. Therefore, the principal response should be to seek to exclude the evidence under s.76 of PACE.

43
Q

You are defending Remington in the Crown Court on a count of s.18 wounding. It is alleged that in revenge for being headbutted on a previous occasion, he repeatedly stabbed Tim in an unprovoked attack. Remington was arrested shortly after the incident and found to be in possession of a knife. He also had traces of blood on his boots. Prior to interview, Remington had been given the opportunity to consult a solicitor.
Throughout the interview under caution, Remington exercised his right to silence. The interviewing officer informed Remington that she believed the knife and blood were attributable to him having stabbed Tim. She asked Remington to provide an explanation for them, but he refused to do so. The interviewing officer did not pursue the matter further and gave no further warnings.
Remington intends to plead not guilty. He instructs you to test the prosecution case and expressly states that he does not wish to give evidence. He asks you whether his refusal to answer questions in interview about the knife and blood can be held against him at trial.
Which response best answers Remington’s question?
[A] An adverse inference cannot be drawn even though Remington refused to explain why he was in possession of a knife and had traces of blood on his boots.
[B] An adverse inference cannot be drawn unless Remington relies on a fact in his defence which he could reasonably have been expected to mention in interview.
[C] An adverse inference cannot be drawn simply because Remington has refused to answer questions in an interview under caution.
[D] Remington cannot be convicted solely on an adverse inference drawn from his refusal to answer the questions.

A

ANSWER
[A] An adverse inference cannot be drawn even though Remington refused to explain why he was in possession of a knife and had traces of blood on his boots.
[A] This is the best answer. Remington was not given the special warning about the potential consequences of his failure to account for the knife and traces of blood during his interview and so no adverse inference can be drawn (s.36(4) CJPOA 1994).
[B] This is not the best answer. This is the test under s.34. The scenario makes it clear that no defence is to be relied on as Remington does not wish to give evidence and instructed counsel to simply test the prosecution’s case.
[C] and [D] whilst correct statements of the law in general, are not tailored to the facts of this case and do not represent the best answers.

44
Q

You represented Preston at his trial in the magistrates’ court where he was convicted of theft of a motorbike. Preston was sentenced to an immediate term of four months’ imprisonment. A month into his sentence, Preston received a letter from a neighbour informing him that she could place the prosecution’s eyewitness at a different location at the time of the alleged theft and this would establish Preston’s innocence. Preston writes to his solicitor stating that he wants to appeal both the conviction and sentence.
What should Preston’s solicitor do now?
[A] Serve the notice of appeal as soon as possible explaining the reason for being out of time.
[B] Apply for an extension of time and then serve notice of appeal within 15 business days.
[C] Advise Preston that he is out of time to appeal against the conviction but that he can still appeal against the sentence.
[D] Make an application for an extension of time along with serving the notice of appeal.

A

ANSWER
[D] Make an application for an extension of time along with serving the notice of appeal.
[A] Is wrong. Crim PR 34.2(1) and (3) requires notice of appeal to be given in writing within 15 business days of sentence being passed. The 15 day time limit has expired. In Preston’s case, an application for an extension of time must be served along with the appeal notice. A simple explanation is insufficient.
[B] Is wrong. By virtue of Crim PR 34.2(3) an application for an extension of time must be served WITH the appeal notice and must explain why the appeal notice is late.
[C] Is wrong. Under Crim PR 34.2(1) and (3) the appellant has 15 business days from the date of sentence to serve notice of appeal, even if that is after the date of conviction where the appeal is against conviction only. The 15 day time limit has expired even since the date that sentence was passed requiring in Preston’s case an application for an extension to appeal out of time
[D] Is correct. The 15 day time limit has expired. By virtue of Crim PR 34.2(3) an application for an extension of time must be served with the appeal notice and must explain why the appeal notice is late.

45
Q

Hannah was arrested on suspicion of theft yesterday. She was interviewed at the police station without a solicitor. She had asked for a solicitor to attend but was told she did not need one as it was ‘all pretty straightforward’ and was encouraged by the police to go ahead with the interview anyway. During the interview Hannah confessed to the theft. She was charged and bailed, and today she has come to see you for advice.
Which of the following statements should you advise Hannah is the correct description of the approach the court will take in deciding whether to exclude her confession evidence under PACE 1984 s.78?
[A] Evidence obtained following any breach of a PACE code will automatically lead to exclusion of that evidence.
[B] If a confession has been obtained after a refusal of access to a solicitor in breach of PACE, the confession is likely to be excluded.
[C] It is necessary to demonstrate that the police have acted in bad faith before the court will exclude a confession obtained in breach of PACE codes.
[D] Provided the evidence has been obtained in good faith by the police, it is unlikely to be excluded by the court, notwithstanding any breaches of PACE.

A

ANSWER
[B] If a confession has been obtained after a refusal of access to a solicitor in breach of PACE the confession is likely to be excluded.
[A] Wrong –there is no automatic exclusion. It is 2-stage process – the court will decide if there has been a breach, and if so whether there has been an adverse effect on the fairness of proceedings. Only if they decide the latter is satisfied will evidence be excluded.
[B] Correct as it is a fundamental right to have access to legal advice and in these circumstances the exclusionary discretion is likely to be exercised in favour of a defendant.
[C] Wrong - there is no requirement to demonstrate bad faith.
[D] Wrong, the test is whether there is an adverse effect on the fairness of the proceedings.

46
Q

You represent Slade, who is on trial in the magistrates’ court for an offence of common assault, said to have taken place in a crowded bar following a football match. A number of prosecution witnesses give evidence at the trial, but the descriptions they give of the assailant are inconsistent. The prosecution have closed their case. You take the view that the identification evidence is very tenuous.
What is your correct course of action at this stage of the trial?
[A] Make a submission of no case to answer at the close of the prosecution case.
[B] Make an application for the proceedings to be stayed on the ground of abuse of process as the prosecution have proceeded with a weak case which they should not have attempted to prosecute.
[C] Call Slade to give evidence denying participation and then make a submission of no case to answer.
[D] Apply to exclude the evidence of the prosecution witnesses under PACE 1984 s.78, pointing out the inconsistencies in the prosecution evidence.

A

ANSWER
[A] Make a submission of no case to answer at the close of the prosecution case.
[A] Correct – although the question does not set out in terms the Galbraith test, it is clear that evidence is tenuous and correct identification of the perpetrator is clearly very relevant to this case, therefore a submission of NCTA would be the most appropriate course of action and this would take place at the close of the prosecution case. Galbraith, Turnbull.
[B] Wrong – although a stay is the appropriate application in an abuse of process situation, this is not an abuse situation.
[C] Wrong. Although a SNCTA is the correct submission, it would not be made after the defendant has given evidence.
[D] Wrong. There is no basis for this application on the facts and any application under s.78 only applies to evidence before it is called.

47
Q

You prosecute Jennifer, who has pleaded not guilty to fraud and consented to summary trial. The complainant is Clarissa, who is 45 years old and has some learning difficulties which mean she has a mental age of approximately 12 years. According to a psychologist’s report, she can understand questions in simple language and give intelligible answers, but will be suggestible and not very robust if she is asked leading questions.
You appear at a pre-trial hearing when you discover that Clarissa and her carer have indicated that they no longer support the prosecution and Clarissa will not be attending court. The police officer in the case wants you to apply for a witness summons.
In these circumstances, will the magistrates issue a witness summons for Clarissa to attend the trial?
[A] Yes, because Clarissa is competent and compellable and so a summons must be granted if the prosecution request one.
[B] Yes, if they are satisfied it is in the interests of justice to issue a summons, because Clarissa is competent and has material evidence to give.
[C] No, because Clarissa is not competent due to her learning difficulties and therefore not compellable.
[D] No, because although Clarissa is competent, a summons should not be used in the case of a vulnerable witness.

A

ANSWER
[B] Yes, if they are satisfied it is in the interests of justice to issue a summons, because Clarissa is competent and has material evidence to give.
[A] Wrong, as it does not properly reflect the provisions of the MCA 1980 s.97 which provide for an interests of justice test.
[B] Correct. Clarissa is likely to be deemed competent as it is her understanding which is important not her robustness. Special measures such as an intermediary might assist. It follows from her competence that she is compellable as obviously she is not a spouse or accused. Therefore, it is a matter of applying the s.97 test.
[C] Wrong. See [B] above.
[D] Wrong. Clarissa’s vulnerabilities may play a part in the magistrates’ application of the test in s.97, but there is no such prohibitive rule and vulnerabilities may well be addressed by special measures .

48
Q

You are prosecuting Angela, who has pleaded not guilty to ABH in a bar which happened at 23:20. Angela’s defence statement sets out that she has been mistakenly identified as the attacker and although she was at the bar earlier in the evening, she had left and had nearly finished walking home by the time of the attack.
At her trial at the Crown Court, Angela gives evidence that on her way home that night, at 23:10, she met Leila and the two of them walked together for 15 minutes towards Angela’s house before separating. Angela calls Leila as a witness, and Leila supports Angela’s account about the time they spent walking together.
Angela’s defence statement does not mention meeting Leila or anyone else after leaving the pub.
You want to ask the judge for sanctions against the defence. What is the appropriate sanction to invite the judge to apply when he directs the jury?
[A] To give less weight to Leila’s evidence because it is not likely that a genuine alibi witness would appear at the last minute.
[B] To draw inferences against Angela as are proper, because the prosecution have not been able to cross-examine Leila effectively.
[C] To disregard Leila’s evidence because Angela failed to set out the details of her alibi in her defence statement.
[D] To draw inferences against Angela as are proper because she failed to set out the details of her alibi in her defence statement.

A

ANSWER
[D] To draw inferences against Angela as are proper because she failed to set out the details of her alibi in her defence statement.

[A] This is not the correct answer. Matters of weight and credibility remain matters for the jury.
[B] This is not the correct answer. Correct sanction, wrong reason. Although some prejudice to the prosecution will often go hand in hand with defects in defence disclosure, in itself it forms no part of the legislative framework of triggers for sanctions.
[C] This is not the correct answer. See the case of Tinnion. The appropriate sanction is not to disregard the evidence but to draw an adverse inference as per [D].
[D] This is the correct answer. As Angela has failed to notify the prosecution of the alibi witness, the jury will be directed that they may draw such inferences as appear proper in deciding whether she is guilty of the offence charged. This encapsulates the requirement to give notice of particulars of fact relied upon, (s.6A CPIA) and the requirement to give notice of particulars of alibi.

49
Q

You prosecute Jade, Steven and Drake, who have been convicted of burglary following a trial in the Crown Court. The evidence against them was that Steven and Drake climbed through an insecure kitchen window into a flat, while Jade remained outside and acted as look out. They were disturbed when the couple who own the flat, Ben and Simon, returned home, at which point all three ran off. When Ben gave evidence at trial, he stated that this was the second time he and his husband, Simon, had been burgled in the past 12 months.
Which is the judge likely to treat as the most aggravating feature?
[A] That the defendants pleaded not guilty and took the case to trial.
[B] That the victims of the offence are a gay couple.
[C] The prevalence of the offence of burglary where Ben and Christian live.
[D] That the defendants committed the offence together.

A

ANSWER
[D] That the defendants committed the offence together.
[A] is wrong. Whilst a plea of guilty will generally attract a reduction in sentence, a plea of not guilty does not aggravate an offence. Lies told by an offender, whether at a police station or during trial, are not an aggravating factor for sentence.
[B] is wrong. Section 66 Sentencing Act 2020 (Aggravation related to sexual orientation) only applies where the offence was motivated by hostility based on orientation or was accompanied by a demonstration of such hostility. The facts suggest this was an opportunistic burglary through an insecure window where the victims were unknown
[C] is wrong. Prevalence is generally not an aggravating feature as it is accounted for in the guidelines. Only in exceptional circumstances (not present here) could a judge take it into account e.g. by evidence supplied by the Criminal Justice Board or emerging from a community impact statement.
[D] is correct. The offenders were operating as in a group.

50
Q

Rihanna is charged with robbery. It is alleged that, together with others, she went into a bank, produced a gun and demanded cash. She was arrested a week later at home and a gun was recovered from the boot of her car. In interview, she denied the offence and suggested that the police had planted the gun.
Before trial, it is disclosed that CCTV from the bank had been deleted in error and the gun, which had been sent for forensic analysis, had been lost. Also, the main prosecution witness had once been arrested as a suspect, but never charged or prosecuted. At the start of the trial, defence counsel made an application for the trial to be stopped. The judge agreed that it was going to be impossible for the defendant to receive a fair trial.
What is the next step that the judge should take?
[A] Direct the prosecution not to proceed because, as a matter of policy, the case should not have been brought.
[B] Balance the competing interests in deciding whether to stop the proceedings for an abuse of process.
[C] Rule that the indictment is defective on the grounds of an abuse of process.
[D] Rule that the indictment should be stayed on the grounds of an abuse of process.

A

ANSWER
[D] Rule that the indictment should be stayed on the grounds of an abuse of process.
[A] is wrong. See DPP v Humphreys (1977): the judge does not have any power to refuse to allow a prosecution to proceed just because she considers as a matter of policy that the case should not have been brought. Also, in this case, there are errors in the evidence being lost and witness issues, but these are not issues which would amount to reasons for the case to have not been brought.
[B] is wrong. No balancing act is required if the Judge concludes the defendant cannot receive a fair trial.
[C] is wrong. The scenario does not raise any issue in relation to the indictment being defective. In any case, a defective indictment does not amount to an abuse of process but is a separate issue.
[D] is correct. The Judge has concluded that the defendant cannot receive a fair trial – a stay in these circumstances is the appropriate step.

51
Q

Bill is arrested on suspicion of attempted burglary. He is found outside a bank in the early hours of the morning. In the wall of the bank is a large hole that has been recently drilled. Bill is covered with dust identical to that from the hole in the wall. He is also carrying a large electric drill. He is arrested and, in the police car on the way to the police station, he makes an admission of guilt to the arresting officer. At the beginning of the interview at the police station, Bill is cautioned and indicates he understands the nature of the caution.
What is the next step that should be taken by the interviewing officer?
[A] Give a special warning and ask Bill to account for his possession of the drill.
[B] Explain to Bill in her own words the nature of the caution.
[C] Give Bill a special warning asking him to account for his presence at the place where he was arrested.
[D] Put to Bill his admission of guilt to the officer in the police car.

A

ANSWER
[D] Put to Bill his admission of guilt to the officer in the police car.
[A] is not correct because the next step at the beginning of the interview after caution is to put the significant statement.
[B] is not correct because Bill has indicated that he understands the nature of the caution.
[C] is not correct because the next step at the beginning of the interview after caution is to put the significant statement.
[D] is correct because the next step at the beginning of the interview after caution is to put the significant statement.

52
Q

Clarke pleaded guilty to s.18 GBH. The victim was his long-term partner, Gemma.
Before passing sentence, based on the serious injuries Gemma sustained, the judge ordered a dangerousness assessment from the probation service and one from a psychiatrist. The psychiatrist reported that, “When domestic tensions arise, Clarke has neither the cognitive nor social skills to understand and articulate his feelings and to problem-solve. These characteristics result in him becoming irrational, impulsive and violent.” The report concluded that Clarke poses a significant risk to members of the public of serious harm occasioned by the commission by him of further specified offences. The pre-sentence report made a similar finding. Clarke has no previous convictions, cautions or reprimands.
Is the judge entitled to make an assessment of dangerousness in relation to Clarke?
[A] No, as Clarke has no previous convictions he cannot be assessed as posing a significant risk to members of the public of serious harm occasioned by the commission by him of further specified offences.
[B] No, as the court must have the opinion of two psychiatrists in order to determine that Clarke poses a significant risk to members of the public of serious harm occasioned by the commission by him of further specified offences.
[C] Yes, the judge is entitled to conclude that Clarke poses a significant risk to members of the public of serious harm occasioned by the commission by him of further specified offences, even in the absence of previous convictions.
[D] Yes, the judge must conclude that Clarke poses a significant risk to members of the public of serious harm occasioned by the commission by him of further specified offences, as both the psychiatrist and probation officer have assessed him as dangerous.

A

ANSWER

[C] Yes, the judge is entitled to conclude that Clarke poses a significant risk to members of the public of serious harm occasioned by the commission by him of further specified offences, even in the absence of previous convictions.

[A] is wrong because it is not necessary for an offender to have previous convictions to be assessed as found a dangerous offender.
[B] is wrong because the judge is entitled to take into account the contents of any relevant report which can include a psychiatric report There is no requirement for there to be any psychiatric report before determining dangerousness, this relates to the making of a hospital order.
[C] is the correct answer. First, the Judge is entitled to take into account the contents of any relevant report which can include a psychiatric report. Second, although it is unusual to assess someone with no previous convictions as dangerous, that is no bar in so concluding. The fact pattern here is extreme and there can be no doubt that this was a serious offence and the Judge, after considering the psychiatric evidence, is entitled to make an assessment of dangerousness in respect of Clarke
[D] is wrong because the Judge has a discretion – they do not have to follow the psychiatrist’s or probation officer’s view but may take the contents of the report into account. Reports before the court are not binding on the sentence (see A above).

53
Q

Martin faces trial for a firearms offence and for robbery. The prosecution allege that Martin, together with an unknown accomplice, went into a post office, produced a gun and demanded money. They then fled the scene with the money. Only Martin was found, arrested and charged. During conference with counsel, it becomes clear that Martin’s defence is duress. He claims that his accomplice had connections to violent gangs and had threatened to kill Martin and his son unless he committed the robbery. In support of his defence, he wishes to rely on three menacing voicemails left on his mobile phone, in which serious threats of violence were made against Martin and his son. The accomplice was never arrested and is not available at the trial.
On what basis is the judge most likely to admit this evidence?
[A] Allow the evidence to be adduced as hearsay evidence under CJA 2003 s.114(1)(d).
[B] Allow the evidence to be adduced as hearsay evidence under CJA 2003 s.116.
[C] Allow the evidence to be adduced as bad character evidence of a non-defendant.
[D] Allow the evidence to be adduced as original evidence.

A

ANSWER
[D] Allow the evidence to be adduced as original evidence.
[D] is the single best answer as the evidence is being adduced for a non-hearsay purpose namely, the state of mind of the hearer, and therefore is not hearsay evidence.
Although [A] and [B] sound plausible they are not because although the evidence of the words do appear to cause another person to believe them, (s115) they are not being adduced by Martin for the purpose of proving them to be true, which is the essence of the definition of hearsay.
[C] Is not the best answer. There is nothing to suggest that a s100 application is to be made or can be made, nor is that the purpose of seeking to admit the voicemails, which is not as bad character but rather the effect of the words spoken on the defendant.

54
Q

You represent Damien, who has been charged with rape. Damien was not on bail at the time of the alleged offence. He has ten previous convictions but has never failed to attend court during any of his previous cases. Six years ago, Damien was convicted of manslaughter and served a three-year custodial sentence. Damien now wishes to make a bail application.
What advice should you give Damien regarding bail?
[A] He cannot be granted bail as he is charged with rape.
[B] He has a prima facie right to bail although in view of the offence with which he is charged it is unlikely that bail will be granted.
[C] He may only be granted bail if the court is of the opinion there are exceptional circumstances which justify it.
[D] He cannot be granted bail as he has a previous conviction for homicide.

A

ANSWER
[C] He may only be granted bail if the court is of the opinion there are exceptional circumstances which justify it.
[A] is wrong as he can be granted bail if there are exceptional circumstances which justify it.
[B] is wrong as he does not have a prima facie right to bail in light of the offence charged and his previous conviction.
[C] is correct as he can be granted bail if the court is of the view that there are exceptional circumstances which justify it. Where the previous conviction was manslaughter, the restriction only applies if the accused received a custodial sentence, which Damien did.
[D] is wrong as this is not the correct test.

55
Q

Julius is charged with s.18 wounding with intent. He is alleged to have deliberately hit Andrew on the head with a glass bottle, causing multiple lacerations to his scalp. In giving evidence at trial, Julius states that he and Andrew were having a fight. He accepts that he hit Andrew with the bottle by mistake, but this was because he had forgotten he had the bottle in his hand, and he never meant to cause Andrew any serious harm. As a result of Julius’ evidence, the judge invites the prosecution to add a count of s.20 wounding as a lesser alternative count. The prosecution decline to do so. The defence agree with the prosecution, and do not wish for the alternative count to be before the jury.
What should the judge do in these circumstances?
[A] Direct the jury only in relation to the one count of wounding with intent, which is on the indictment.
[B] Insist that the Prosecution add the count of s.20 wounding to the indictment.
[C] Direct the jury on returning an alternative verdict of s.20 wounding as, in the circumstances, it would be in the interests of justice to do so.
[D] Direct the jury on returning an alternative verdict of s.20 wounding as it has been raised by the Defendant.

A

ANSWER
[C] Direct the jury on returning an alternative verdict of s.20 wounding as, in the circumstances, it would be in the interests of justice to do so.
[A] Wrong. The judge can direct the jury on an alternative verdict even if it is not explicitly set out on the indictment.
[B] Wrong. The judge cannot insist that the prosecution add a further count, but is able to direct the jury on an alternative count in any event
[C] Is the right answer. The judge is not obliged to direct the jury on finding the defendant guilty of an alternative offence, even if it is available to them. However, if the possibility that the accused is guilty of a lesser offence has been obviously raised in evidence, the judge should, in the interests of justice, leave the alternative to the jury in the interests of justice, even if the prosecution and defence counsel do not wish for it to be done.
[D] Wrong. The judge is not under an obligation to direct the jury because the s.20 wounding has been raised by the defence in evidence; the key issue is instead whether it is in the interests of justice to do so, as in [C] above.

56
Q

Millie (aged 16) appears before the youth court charged with an offence of causing s.20 GBH (which, in the case of an adult, carries a maximum sentence of five years’ imprisonment). The defence have been served with initial details of the prosecution case and Millie intends to plead not guilty. She has never been in trouble with the police before and the magistrates do not consider her to be a dangerous offender.
Which procedure will the magistrates adopt to determine venue for Millie’s trial?
[A] The magistrates must conduct a plea before venue procedure before deciding where Millie should be tried.
[B] Millie should be sent forthwith to the Crown Court for trial.
[C] Millie’s plea should be taken, and the matter adjourned for trial in the youth court.
[D] Millie should be asked in which court she wishes to be tried.

A

ANSWER
[C] Millie’s plea should be taken, and the matter adjourned for trial in the youth court.
[A] Wrong because the plea before venue procedure would only apply in the circumstances envisaged by s.24A MCA. Millie would have to be charged with a grave offence which might attract a sentence under s.249-252 Sentencing Act 2020 (formerly 91 PCC(S)A 2000) (an offence carrying at least 14 years’ imprisonment in the case of an adult).
[B] Wrong because where a juvenile appears in the youth court she can only be sent to the Crown Court for trial in the circumstances set out in s.51A(3) CDA and s.24A MCA. None of those circumstances apply: in particular, she is not a dangerous offender, and this is not a grave offence (as above).
[C] Correct. [C] is correct because as this case must be tried in the youth court: the same procedure is adopted as for a summary trial in the magistrates’ court.
[D] Wrong because a juvenile has no right to elect a Crown Court trial. The normal rules governing allocation (mode of trial) do not apply in the case of juveniles.

57
Q

Last week, Liam appeared in the magistrates’ court charged with s.18 GBH for attacking PC Khan on 14 February. The matter was sent to the Crown Court. Liam’s cousin, Nayen, now appears before the magistrates’ court charged with threatening behaviour contrary to the Public Order Act 1986 s.5 (a summary only offence with a maximum sentence of a fine) and assault with intent to resist arrest (an either-way offence). These charges relate to his conduct on 14 February and arise from Liam’s attack on PC Khan. Nayen intends to plead not guilty in respect of both matters.
You are representing Nayen, who instructs you that he would prefer to be tried in the magistrates’ court, but wants to know if it is likely he will be tried in the Crown Court alongside his cousin. In your assessment, the magistrates will almost certainly conclude that Nayen’s alleged offences are related to Liam’s GBH charge already sent to the Crown Court.
What is the best advice to give Nayen?
[A] Nayen must be tried in the Crown Court for the charge of assault with intent to resist arrest.
[B] It is possible that Nayen will be tried in the magistrates’ court for both the assault with intent to resist arrest and s.5 charges.
[C] It is most likely that Nayen will be tried in the magistrates’ court for the charge of threatening behaviour under s.5.
[D] It is most likely that Nayen will be tried in the Crown Court on the assault with intent to resist arrest charge but must be tried in the magistrates’ court on the s.5 threatening behaviour charge.

A

ANSWER
[D] It is most likely that Nayen will be tried in the Crown Court on the assault with intent to resist arrest charge but must be tried in the magistrates’ court on the s.5 threatening behaviour charge.
[A] Not the best answer. Whilst it seems likely that Nayen will be sent to the Crown Court for the assault with intent to resist arrest charge, it is not certain. This is because the decision is at the magistrates’ discretion due to the GBH charge having been sent at an earlier hearing (s.51(3) & (4) CDA 1998.
[B] This is not the best answer. Whilst technically true, it is not the most likely outcome on the facts of this scenario and therefore not the best advice – see D.
[C] Not the best answer. The section 5 charge will have to be tried in the magistrates’ court because it is a summary only offence for which imprisonment and/or disqualification from driving cannot be imposed and so it cannot be sent to the Crown Court.
[D] This is the best answer. It is most likely that Nayen will be sent to the Crown Court for the assault with intent to resist arrest charge because it is intrinsically linked to Liam’s GBH charge. However, the section 5 charge cannot be sent to the Crown Court because it is a summary only offence for which imprisonment and/or disqualification from driving cannot be imposed.

58
Q

Joshua is charged with ABH on his mother, Violet. Violet’s witness statement says that Joshua argued with her in the kitchen, and as she reached for something in a cupboard, he slammed the cupboard door onto her arm, causing a broken wrist.
In his police interview, Joshua says he did slam the door but contact with Violet’s arm was an accident as she reached out unexpectedly. There is medical evidence of Violet’s broken wrist, which is not disputed. There are no other witnesses.
Two weeks before the trial, Violet writes to the prosecution, stating that she no longer wishes to give evidence against Joshua as he is dealing with his anger issues. This letter also states that Violet does not believe she is well enough to attend court due to a heart condition, and the letter is accompanied by her recent hospital appointment letters referring to heart problems. There is no further contact between Violet and the prosecution.
On the day of the trial, Violet does not attend court and efforts to contact her fail. Prosecution counsel seeks to persuade the judge that Violet’s statement about the assault should be admitted under s.114(1)(d) of the Criminal Justice Act 2003.
What is the judge most likely to do?
[A] Admit the statement because there is no reason to doubt its reliability, and it is supported by the injury evidence and to some extent Joshua’s account.
[B] Admit the statement because it has probative value and there is evidence that Violet has a good reason for not attending court.
[C] Exclude the statement because, while it is not the sole evidence against Joshua, it is decisive evidence and prejudicial to Joshua.
[D] Exclude the statement because this gateway is to be applied with caution, and should not be used to circumvent the most appropriate gateway for admission.

A

ANSWER
[D] Exclude the statement because this gateway is to be applied with caution, and should not be used to circumvent the most appropriate gateway for admission.
[A] Not the best answer. Reliability and importance of the evidence, and the question of what other evidence there is on the matter, are factors to be considered (CJA 2003 s.114(2), but here they are not strong enough reasons to lead to admissibility as although the factual issues are narrow, Violet’s statement is the only evidence on the question of accident and factor (g) would play an important role, as in Tindle where the disputed evidence was that of the complainant and the prosecution had not done enough to secure their attendance.
[B] Not the best answer. Where a s.116 gateway is invoked, it is generally wrong to admit the evidence under s114(1)(d) if the conditions of that gateway would not be satisfied (approach of D(E) and Horncastle. The probative value of such evidence would not lead to admissibility. Here, more/better evidence that Violet is unfit to give evidence would be required
[C] Not the best answer. These are relevant considerations, albeit ‘sole’ and ‘decisive’ do not reflect the wording used in the s.114(1)(d) factors, but do not get to the heart of why the evidence would be excluded.
[D] The best answer. Similarities with D(E) and Tindle suggest that factor (g) (the fact that oral evidence of the matter could have been given) and the actions of the prosecution would be fatal to admissibility. The prosecution should not seek to circumvent their own failure to render the statement admissible under s.116.

59
Q

Suzi was arrested last night for attempting to murder her wife, Melanie, who was found lying on the floor of their home in an unconscious state. It appears that she had been hit on the head with a heavy, blunt object.
Suzi and Melanie’s neighbour, Jed, told the police that shortly after Melanie was taken away by ambulance, Suzi came to his house. When Jed asked her about Melanie, Suzi told him that during an argument earlier that evening she had pushed Melanie, which caused her to lose balance and fall over, banging her head. Suzi said that she had not intended to hurt Melanie. She also told Jed that Melanie was a jealous and controlling partner.
You are the duty Crown Prosecutor and need to make a charging decision. The police have asked you about the significance of Jed’s evidence.
What is the correct advice to give about what Suzi said to him?
[A] It is inadmissible because it was not made to the police.
[B] It is a mixed statement and constitutes a confession.
[C] It is exculpatory and is not a confession.
[D] It is admissible because it is relevant.

A

ANSWER
[B] It is a mixed statement and constitutes a confession.
[A] This answer is wrong because a confession can be made to anyone, including friends, and is not limited to statements made to a person in authority. (PACE 1984, s.82(1)).
[B] This is the correct answer. A confession may be ‘wholly or partly adverse’ to the maker, with the result that a so-called ‘mixed statement’, which is part confession and part exculpation, is a confession for the purposes of the PACE 1984. The fact that Suzi acknowledges she pushed Melanie is partly adverse, even if the rest of the statement (that she had not intended to hurt Melanie and her injuries were an accident) is exculpatory.
[C] This is wrong because it is a mixed statement because while the majority of her statement was exculpatory, it was also partly adverse. See the explanation to [B] above.
[D] This is wrong. It does not go far enough to satisfy the test that it is admissible as relevant confession evidence. Although this is correct, and evidence of confession is admissible if it is relevant under PACE 1984, s.76(1) . This is not the best answer on the facts provided, specifically given the stage of proceedings at which you are being asked to advise.

60
Q

You are instructed to represent Roberta at her forthcoming trial in the magistrates’ court. Roberta is charged with taking a vehicle without consent.
Your instructing solicitor confirms to you that the prosecution have complied with their initial duty of disclosure and they have served the unused material on the defence today.
What would be your advice in relation to the defence duty of disclosure?

[A] They must now serve the defence statement within 28 days.
[B] They must now serve the defence statement within 14 days.
[C] They are under no obligation to serve a defence statement.
[D] They must now serve the defence statement within 21 days.

A

ANSWER

[C] They are under no obligation to serve a defence statement.

[A] This answer and time limit is wrong. It refers to cases sent to the Crown Court.
[B] This answer is wrong. There is no duty on the defence to serve a defence statement because Roberta is charged with a summary offence. However, if the defence elects to serve a defence statement, it must be served within 14 days.
[C] This is the correct answer. In cases tried summarily there is no obligation on the defence to provide a defence statement.
[D] This answer is wrong and fictitious. It is neither the correct time limit for cases tried summarily or on indictment.

61
Q

James is awaiting trial for fraud in the magistrates’ court. He is on conditional bail with conditions of residence at his home address and a surety in the sum of £500. His mother, Margaret, is the named surety. Margaret suspects that James will not attend his trial. She writes to the officer in the case to indicate that James is unlikely to surrender to custody, and she no longer wishes to stand as surety. As a result of this, the officer arrests James at 15:00 on Saturday. James is held in custody until his appearance at court on Monday morning.
You are briefed to attend at James’s breach of bail hearing at 10:00 on Monday.
Which of the following is the correct advice to give James before the hearing?
[A] As his mother gave written notice to the police that James is unlikely to surrender to custody, the police acted within their powers by arresting him.
[B] The Court is obliged to extend James’s bail as the statutory 24-hour time limit for putting him before the court, following his arrest, has now expired.
[C] As his mother is no longer willing to stand as surety, James must be remanded in custody.
[D] As his mother is no longer willing to stand as surety, if the court is to grant bail, then more onerous conditions must be imposed.

A

ANSWER
[A] As his mother gave written notice to the police that James is unlikely to surrender to custody, the police acted within their powers by arresting him.
[A] is correct. Margaret has complied with s.7(3) Bail Act 1976 by giving written notice that James is unlikely to surrender and that she wishes to be relieved of any obligations, therefore the police have the power to arrest James under s.7(3).
[B] is wrong. The maximum time period is 24 hours but Sundays are specifically excluded.
[C] is wrong. The court has the discretion to remand in custody, or grant bail on the same or different conditions.
[D] is wrong. There is no rule that more onerous conditions need to be imposed (although they generally are if bail is being granted under s7(5)).

62
Q

You represent Travis, who has been convicted of common assault on his ex-girlfriend, Sophie. Travis went to Sophie’s house at approximately 14:00 and demanded to be let in. When Sophie refused to let Travis in, he attempted to force his way past her, knocking Sophie to the floor. Travis has three recent previous convictions for criminal damage. The first for damaging Sophie’s car, which was parked outside her home address. The second for breaking the living room window at Sophie’s home, and the third for damaging the front door at Sophie’s mother’s home, which is next door to Sophie’s house. The three previous offences took place on different dates at around midnight, when Travis had been drinking in a nearby bar.
The court has requested a pre-sentence report and is considering imposing a community order. If the court does so, which one of the following requirements is the court most likely to impose?
[A] An alcohol treatment requirement
[B] A residence requirement
[C] An exclusion requirement
[D] A curfew requirement

A

ANSWER
[C] An exclusion requirement
[A], [B] and [D] Are not the most appropriate answers because an exclusion requirement is the most appropriate requirement in the circumstances because the current offence, and recent previous convictions, took place in a specified area where a specified person, Sophie, (and her mother) live.
[C] Is the most appropriate answer because the current offence, and recent previous convictions, took place in a specified area where a specified person, Sophie, (and her mother) live.

63
Q

Tim and Ryan have been charged with ABH on Nafis. Ryan is alleged to have held Nafis while Tim punched him. The prosecution will be calling three witnesses to the offence, each of whom gives a slightly different account. One of them describes Nafis as acting aggressively towards Tim immediately prior to the alleged assault. Both Tim and Ryan gave ‘no comment’ interviews.
At trial, Tim’s case will be self-defence. Ryan will admit being present at the incident but will deny holding Nafis or taking any part. Ryan will describe how Tim, having been attacked by Nafis, punched Nafis once in order to defend himself. Tim does not wish to give evidence.
How can the evidential burden in relation to self-defence be discharged?
[A] The evidential burden can only be discharged by Tim himself giving evidence of self-defence.
[B] The evidential burden can be discharged if Ryan gives evidence that Tim acted in self-defence.
[C] The evidential burden cannot be discharged by a prosecution witness giving evidence that Tim acted in self-defence.
[D] The evidential burden can be discharged by evidence from a prosecution witness, but only if that evidence is adduced by the defence in cross-examination.

A

ANSWER
[B] The evidential burden can be discharged if Ryan gives evidence that Tim acted in self-defence.
[B] is correct. In relation to self-defence Tim will bear the evidential burden, as the question indicates. Although the evidential burden is borne by Tim, the burden is discharged wherever there is sufficient evidence of the defence, no matter how or by whom that evidence is adduced. It follows that [B] is correct and the other answers are wrong.

64
Q

Frederick faces one count of arson. One of the original jurors in his trial was discharged so the jury now comprises 11 members.
In which of the following circumstances is the judge able to accept the jury’s verdict?
[A] When nine of the jurors are in agreement after two hours and ten minutes of deliberation.
[B] When, after three hours of deliberation, the jury notify the judge that ten of them are in agreement.
[C] When, after one hour and 30 minutes, the jury notify the judge that they wish to return a ten to one majority verdict.
[D] As there are now 11 jurors, the judge can only accept a unanimous verdict, which can be given at any time.

A

ANSWER
[B] When, after three hours of deliberation, the jury notify the judge that ten of them are in agreement.
[A] Wrong as although a majority direction can be given after 2 hours and 10 minutes, at least ten of the jurors must agree.
[B] Is correct as there must be at least 10 agree for a majority verdict. The majority direction can be given at any time after 2 hours and 10 minutes have passed.
[C] Wrong. This is insufficient time for a majority direction to be given.
[D] Wrong. The judge can accept a majority verdict from ten jurors.

65
Q

Nathan and his brother Ross went to a bar to watch a football match on TV. A group of opposing supporters were also watching the match in the bar. When the opposing team scored, Nathan and Ross attacked one of the opposing supporters (male 1); Nathan put him in a head lock so that Ross could punch him in the face. A few metres away, Damien, who was unknown to Nathan and Ross, punched another rival supporter in the face (male 2). Nathan, Ross and Damien were all detained at the scene. The police took witness statements from a number of people who had witnessed both assaults.
The two men that Nathan, Ross and Damien attacked each sustained injuries amounting to ABH. All three defendants accept that they were present at the bar, but each denies participation in any attack. The CPS has instructed you to draft the indictment.
What is the best way of indicting these defendants?
[A] Draft one indictment. On Count 1, Ross should be charged with ABH on male 1 and Nathan charged with aiding and abetting him. On Count 2, Damien should be charged with ABH on male 2.
[B] Draft one indictment. On Count 1, Nathan and Ross should be jointly charged with ABH on male 1. On Count 2, Damien should be charged with ABH on male 2.
[C] Draft two indictments. In Indictment 1, Nathan and Ross should be jointly charged with ABH on male 1. In Indictment 2, Damien should be charged with ABH on male 2.
[D] Draft two indictments. In Indictment 1, Ross should be charged with ABH on male 1 and Nathan charged with aiding and abetting him. In Indictment 2, Damien should be charged with ABH on male 2.

A

ANSWER
[B] Draft one indictment. On Count 1, Nathan and Ross should be jointly charged with ABH on male 1. On Count 2, Damien should be charged with ABH on male 2.

[A] This is not the best answer - Nathan and Ross should be indicted in a single count and that there is no need to distinguish between principal offenders and secondary parties.
[B] This is the best answer. Nathan and Ross are alleged to have jointly attacked male 1 and so may be properly joined in a single count. As the charges against all men arise out of the same incident, there is a proper basis for drafting a single indictment, notwithstanding that Damien is accused of assaulting man 2.
[C] and [D] are not the best answers. Though there is no joint count linking Ross and Nathan with Damien, where the offences separately alleged against the accused are, on the evidence, so closely related by time or other factors, the interests of justice are best served by a single trial. Here the attacks occurred at the same time, during the same event, apparently precipitated by the same incident (the goal), and witnessed by the same people (who would have to attend court twice if there were separate trials).

66
Q

Neal is charged with ABH. It is alleged he assaulted Kane in a nightclub. When interviewed by the police, Neal denied he had been there on the night of the assault. In evidence at trial, he accepted that he had lied about this because he did not want his wife to find out he had been in the nightclub. Neal had promised his wife that he would not go there, as he had recently had an affair with one of the bar staff.
What direction should the judge give to the jury in relation to Neal’s lie?
[A] The jury may not rely on Neal’s lie to support evidence of his guilt.
[B] The jury must consider whether there was an innocent motive for Neal’s lie before being able to rely on the lie to support evidence of his guilt.
[C] The jury may rely on Neal’s lie to support evidence of his guilt.
[D] The jury must not rely on Neal’s lie as evidence of his guilt as he has provided an explanation for it.

A

ANSWER
[B] The jury must consider whether there was an innocent motive for Neal’s lie before being able to rely on the lie to support evidence of his guilt.
[A] Is not the correct answer as the jury may rely upon the lie as supporting evidence of Neal’s guilt if they are sure he did not lie for an innocent reason.
[B] Is the correct answer as the jury may rely upon the lie as supporting evidence of Neal’s guilt if they are sure he did not lie for an innocent reason.
[C] Is not the correct answer as the jury may only rely upon the lie as supporting evidence of Neal’s guilt if they are sure he did not lie for an innocent reason.
[D] Is not the correct answer as the jury may rely upon the lie as supporting evidence of Neal’s guilt, even where he has provided an explanation, if they are sure he did not lie for an innocent reason.

67
Q

You defend Samira, who has been charged with s.20 wounding and sent to the Crown Court for trial. It is alleged that Samira approached a stranger at a bus stop and slashed him with a knife. Your solicitor has obtained a medical report from Dr Woods, who is an approved registered medical practitioner under the Mental Health Act 1983. Dr Woods concludes that Samira is not fit to stand trial. Your solicitor asks for clarification about the procedure which will be followed in the Crown Court.
What is the correct advice to give him?
[A] A jury will determine whether Samira is fit to be tried on hearing evidence from Dr Woods and another registered medical practitioner.
[B] A judge will determine whether Samira is fit to be tried on hearing evidence from Dr Woods alone.
[C] A judge will determine whether Samira is fit to be tried on hearing evidence from Dr Woods and another registered medical practitioner.
[D] A jury will determine whether Samira is fit to be tried on hearing evidence from Dr Woods alone.

A

ANSWER
[C] A judge will determine whether Samira is fit to be tried on hearing evidence from Dr Woods and another registered medical practitioner.
[A], [B] and [D] Are not correct. The Criminal Procedure (Insanity) Act 1964, s. 4(1), 4(5), 4(6) provides that the question of fitness to be tried shall be determined by a judge/court without a jury and no determination shall be made except on the written or oral evidence of two or more registered medical practitioners, at least one of whom is duly approved.
[C] Is correct. See answer explanation above.

68
Q

You are defending Nick, who is charged with rape. Nick suffers from anxiety and depression. On the second morning of the trial, Nick fails to attend, and enquiries made by your instructing solicitor suggest that Nick has left the country. The trial continues and you have sufficient instructions to continue to represent Nick in his absence.
The prosecution call the rest of their evidence including reading out a summary of Nick’s police interview under caution, during which he denied the allegations and explained his defence. After the prosecution close their case, the judge asks you if you wish to call any evidence. You decline and indicate that you are ready to deal with the directions that should be given to the jury. The judge tells you that she is minded to direct the jury that they are entitled to draw adverse inferences against Nick because of his failure to give evidence at trial.
What is the best submission for you to make in response?
[A] The jury should not be permitted to draw an adverse inference because to do so would be unfair in the circumstances of the case.
[B] The jury should not be permitted to draw an adverse inference because they cannot rule out the possibility that Nick’s mental condition would have made it undesirable for him to give evidence.
[C] The jury should not be permitted to draw an adverse inference in this case because the defendant absconded before he could be told by the court that he was entitled to give evidence and warned of the consequences of failing to do so.
[D] The jury should not be permitted to draw an adverse inference in this case because an inference in relation to failing to testify cannot be drawn against a defendant who has absconded from his trial.

A

ANSWER
[C] The jury should not be permitted to draw an adverse inference in this case because the defendant absconded before he could be told by the court that he was entitled to give evidence and warned of the consequences of failing to do so.
[A] is not the best answer. While the judge does retain a discretion to advise a jury against drawing an adverse inference against a defendant who has failed to testify, such a direction requires, “either ‘some evidential basis for doing so or some exceptional factors in the case making that a fair course to take’.” Here the justification for directing the jury against drawing the inference is the failure to comply with the condition set out in s.35(2).
[B] is not the best answer. A finding under s. 35(1)(b) would be an appropriate basis on which to direct the jury not to draw an adverse inference. However there has to be an evidential basis for such a finding. This scenario is silent as to whether any evidence has been called on behalf of Nick about his mental condition or anything else. Further he was capable of giving a full account in interview, one of the factors which can militate against a finding under s. 35 (1)(b).
[C] is the best answer. The court’s obligation in s. 35(2) to satisfy itself that the accused knows that he can, if he wishes, give evidence is mandatory and cannot be overlooked even where the accused has, by absconding, put himself beyond the reach of the warning. The defendant’s decision whether to give evidence is probably the most significant decision which a defendant has to make in the course of his criminal trial. For the purposes of the requirement in s35(2) The court must satisfy itself that the accused is aware of the possible consequences of a decision not to testify. The facts of this case suggest that Nick absconded before the necessary warnings could be given and consequently this is the best argument against permitting the jury to draw an adverse inference against him.
[D] is not the best answer. This answer overlooks the possibility that a defendant could abscond after he has been given the warning in section 35(2). In such a scenario an adverse inference based on a failure to testify could be drawn.

69
Q

You defend Adam, who is charged with ABH on his girlfriend, Katie. He pleaded not guilty and the case was adjourned for trial in the magistrates’ court. He was bailed on three conditions: (1) to live at his parents’ house; (2) not to contact his girlfriend; and (3) not to go within 500 metres of her address.
On Thursday evening at 21:00, Katie called the police to report that Adam was outside her house. He was arrested and is being produced before the magistrates on Friday morning.
Adam instructs you that he was going to Katie’s house (for which he has a key) to pick up some of his clothes, as the only clothes he had were the ones he was wearing when arrested. He was sure Katie would be out as she usually attends yoga on Thursday evening.
What is the best advice to give Adam as to what will happen in court?
[A] The court will remand Adam in custody until his trial as he has breached his bail.
[B] The court will hear evidence as to whether or not the bail condition has been breached. If the prosecution proves the breach, the court will consider whether to impose more stringent conditions on Adam’s bail.
[C] The court will hear representations as to whether or not a breach has occurred. If the court decides that there has been a breach it will go on to consider whether Adam should be granted bail again.
[D] The court will hear representations as to whether or not Adam had a reasonable excuse for breaching his bail condition. If he did, the court will re-admit him to bail on the same conditions as before.

A

ANSWER
[C] The court will hear representations as to whether or not a breach has occurred. If the court decides that there has been a breach it will go on to consider whether Adam should be granted bail again.
[A] Is not the best answer. Whilst it may turn out to be correct, this advice conveys the impression that the court will automatically remand Adam in custody which is not necessarily the case as they may grant bail subject to the same or different and or more onerous conditions.
[B] Is not the best answer as the court need not hear evidence; instead it can base its decision on representations from the prosecution and defence. The advice also gives the impression that Adam will automatically be re-admitted to bail, albeit on more stringent conditions which is not necessarily the case (see [A]).
[C] Is the best answer. The court is unlikely to hear evidence on the question of breach and is more likely to resolve the matter on representations. In deciding this, the court will adopt a two-stage approach. First, the court will determine whether there has been a breach of a bail condition; second, if there has been a breach, the court will consider whether Adam should be bailed again; this does not include an enquiry as to whether Adam had a reasonable excuse for breaching bail..
[D] Is not the best answer as it suggests that a reasonable excuse will totally exonerate Adam, which, for the reasons given, is not correct. The court will hold an enquiry as to whether Adam has breached his bail. The reasons for breaching bail only come into play when deciding whether to re-bail Adam.

70
Q

Lukasz has been arrested by PC Briggs on suspicion of handling stolen goods. In arresting Lukasz, PC Briggs approached him from behind and handcuffed him. PC Briggs then told Lukasz that he was under arrest for handling stolen goods.
What justification could PC Briggs give for the use of handcuffs in effecting this arrest?
It was reasonably necessary if PC Briggs intended to
[A] prevent Lukasz from disposing of the stolen goods.
[B] prevent Lukasz from escaping.
[C] arrest Lukasz.
[D] convey Lukasz to the police station.

A

ANSWER
[B] prevent Lukasz from escaping.
[B] is correct - Handcuffs should only be used where they are reasonably necessary to prevent an escape or to prevent a violent breach of the peace by a prisoner.
[A], [C] & [D] are all wrong.

71
Q

You prosecute Ian, who is charged with possession with intent to supply Class A drugs. George (aged 18), who is to be called as a witness to the offence, has a learning disability which affects his ability to communicate clearly. His parents tell you that George is anxious about testifying as he is worried about repercussions from Ian’s family if he gives evidence. You wish to apply for a special measures direction for George.
Is George eligible for a special measures direction?
[A] Yes, he will be eligible due to his learning disability if it is likely to diminish the quality of his evidence.
[B] Yes, he is eligible by virtue of his age.
[C] Yes, he is automatically entitled due to his learning disability.
[D] Yes, he is automatically entitled if he is in fear of testifying.

A

ANSWER
[A] Yes, he will be eligible due to his learning disability if it is likely to diminish the quality of his evidence.
[A] Is correct - he is entitled to special measure due to his disability if the quality of his evidence is likely to be diminished.
[B] Is wrong – he is not entitled to special measures due to his age as he is not under 18.
[C] is wrong – he is only entitled to special measures in these circumstances as per [A] above
[D] is wrong – fear also requires that the quality of his evidence is likely to be diminished.

72
Q

You represent Kyle, who faces a single count of violent disorder (an either-way offence). You meet Kyle in conference a week before the PTPH and he accepts that he is guilty of the offence but vehemently disputes some aspects of the prosecution case. You take the view that the differing accounts are very likely to make a difference to the ultimate sentence in this case.
What is the most appropriate advice to give Kyle about the approach that should be taken?
[A] Plead not guilty as he does not accept the prosecution case.
[B] Plead guilty on the basis of the prosecution case because he accepts that he is guilty of the offence.
[C] Prepare a written basis of plea based on his account and invite the prosecution to agree it.
[D] Seek to have the matter listed for a Newton hearing to resolve the differences between the prosecution case and his account.

A

ANSWER
[C] Prepare a written basis of plea based on his account and invite the prosecution to agree it.
[A] Is not the most appropriate answer as Kyle accepts he is guilty of the offence and the appropriate approach is to notify the prosecutor of the areas of difference. The prosecutor may agree Kyle’s version of events and subject to acceptance by the judge this will be the basis upon which he is sentenced.
[B] Is not the most appropriate answer as the more appropriate approach is to notify the prosecutor of the areas of difference. The prosecutor may agree Kyle’s version of events and subject to acceptance by the judge this will be the basis upon which he is sentenced.
[C] Is the most appropriate answer as the prosecutor may agree Kyle’s version of events and, subject to acceptance by the judge, this will be the basis upon which he is sentenced.
[D] Is not the most appropriate answer as the prosecutor may agree Kyle’s version of events and subject to acceptance by the Judge this will be the basis upon which he is sentenced. Whilst the case may need to be listed for a Newton hearing this is not the most appropriate initial approach as the prosecution may accept the basis of plea.

73
Q

You represent Abigail, who is charged with ABH after attending a party where a fight broke out. At trial, the magistrates are of the view that although there is insufficient evidence to prove Abigail is guilty of the offence, there is evidence that she was nonetheless involved in the fight.
You are the court legal adviser. What advice should you give the magistrates about their decision?
[A] The court may decide that Abigail is guilty of a lesser offence of common assault and, if it decides to find her guilty of the lesser offence, it must give sufficient reasons to explain its decision.
[B] The court may decide that Abigail is guilty of another specified offence of violence and, if it decides to find her guilty of the lesser offence, it must give its reasons in the form of a judgment.
[C] The court may not convict Abigail of another or lesser offence but must give detailed reasons to explain its decision.
[D] The court may not convict Abigail of another or lesser offence and may, but is not obliged to, provide an explanation for its decision.

A

ANSWER
[D] The court may not convict Abigail of another or lesser offence and may, but is not obliged to, provide an explanation for its decision.
[A] Although it is true that the court, when convicting, must give sufficient reasons to explain its decision, the court may not find that Abigail is guilty of the lesser charge of common assault because she was not separately charged with it.
[B] This answer is wrong because the court, when convicting, is not required to state their reasons in the form of a judgment or to give reasons in any elaborate form, and also the court may not convict Abigail of any other offence – see the answer to [D] in more detail below.
[C] This answer is only partially correct because although it is accurate that the court may not convict Abigail of any other offence, the magistrates do not have to give any reasons if they acquit. In any event, whilst reasons must be given in the event of a conviction, they need to be “sufficient” but do not need to be detailed.
[D] This is correct because “The justices are restricted to reaching a decision of guilty or not guilty on the charge actually before them. They have no power to find an accused not guilty as charged but guilty of a lesser offence (Lawrence v Same [1968] 2 QB 93). Unlawful wounding is not an offence for which an accused may be found guilty of a lesser offence (compared to certain driving offences and aggravated vehicle taking). Abigail was only charged with one offence, and not two separate charges / alternative counts. Also, the court is not under a duty to provide reasons - if the court acquits the accused, it may (but is not required to) give an explanation of its decision (Crim PR 24.3(6)(a)).

74
Q

In which circumstances can the magistrates commit an offender to the Crown Court for sentence?
[A] The magistrates can commit an offender convicted of any offence to the Crown Court for sentence provided they are satisfied that their sentencing powers are inadequate.
[B] The magistrates can commit an offender convicted of any either-way offence to the Crown Court for sentence provided they are satisfied that their sentencing powers are inadequate.
[C] The magistrates can only commit an offender convicted of any either-way offence to the Crown Court for sentence if the offender has entered a guilty plea.
[D] The magistrates can commit an offender convicted of any either-way offence to the Crown Court for sentence provided the interests of justice test is satisfied.

A

ANSWER
[B] The magistrates can commit an offender convicted of any either-way offence to the Crown Court for sentence provided they are satisfied that their sentencing powers are inadequate.
[A] Is wrong because the magistrates only have the power to commit one or more either-way offences to the Crown Court for sentence under s.14 or s.18 SA 2020, not ‘any’ offence.
[B] Correct because of the explanation/authority in [A] above. The magistrates can commit one or more either-way offence to the Crown Court where the court takes the view that the seriousness of the offence(s) is such that its sentencing powers are inadequate.
[C] Wrong because this is not strictly accurate since it refers to ‘any offence’ and not either-way offences and furthermore the magistrates can commit for sentence an either-way offence after they have convicted a defendant after summary trial.
[D] Wrong since there is no such test. The correct test is that in [B] above.

75
Q

Jenna was tried by a jury and convicted of burglary. 14 days later she was sentenced to three years’ imprisonment for that crime. She wishes to appeal against her conviction.
When must Jenna give notice of application for leave to appeal to the Court of Appeal?
[A] Within 28 days of her conviction
[B] Within 28 days of her sentence
[C] Within 21 days of her conviction
[D] Within 21 days of her sentence

A

ANSWER
[A] Within 28 days of her conviction
[A] Correct. Notice of application for leave to appeal against conviction must be lodged within 28 days of conviction. Should the trial judge have certified that the case is fit for appeal, then the notice of appeal based on that decision should similarly be lodged within the same time period.
Therefore [B], [C] and [D] are wrong.